NOTEPAD
Results
of 200 questions answered correctly

You have reached of 200 points, ( %)

Your time

Question 1 of 200

1. A 35-year-old man has been delivered into a surgical ward with a suppurating wound in the neck, anterior to the trachea (previsceral space). If a surgical operation is not performed urgently, there is a risk of infection spreading to the:

Explanation

The mediastinum is the space lying between the right and left pleurae of the lungs in the chest. It extends from the posterior aspect of the sternum to the anterior surface of the thoracic vertebrae and contains all of the thoracic viscera except the lungs. It is divided into three major parts: anterior, middle, and posterior. The anterior mediastinum is bound above by the thoracic inlet, laterally by the pleura, anteriorly by the posterior surface of sternum, and posteriorly by the pericardium and great vessels. It contains loose areolar tissue, lymph nodes, some lymphatic vessels that ascend from the convex surface of the liver, thymus, thyroid, parathyroids, and internal mammary arteries and veins. It is termed "previsceral space". Although, the trachea is in the middle mediastinum but the wound was anterior to it, therefore, it can easily spread to the anterior mediastinum. (Visceral means organs) The middle mediastinum, the "visceral space," contains the heart and pericardium, the ascending and transverse arch of the aorta, the superior vena cava and the azygos vein that empties into it, the brachiocephalic arteries and veins, the phrenic nerves, the upper vagus nerves, the trachea and its bifurcation, the main bronchi , the pulmonary artery and its two branches, the pulmonary veins, and adjacent lymph nodes. It is bound in front by the anterior mediastinum and posteriorly by the posterior mediastinum. The posterior mediastinum, lies behind the heart and pericardium and extends from the level of the thoracic inlet to the twelfth thoracic vertebra. It contains the thoracic portion of the descending aorta, esophagus, thoracic duct, azygos and hemiazygos veins, lymph nodes, sympathetic chains, and inferior vagus nerves. 
2. In the life cycle of a cell during mitosis a natural change in the amount of genetic material occurs. The DNA doubles at the following stage:

Explanation

Interphase is the phase of the cell cycle in which a typical cell spends most of its life. During this phase, the cell copies its DNA in preparation for mitosis or meiosis. During Interphase, cellular organelles double in number, the DNA replicates, and protein synthesis occurs. Without Interphase there is no possible way a cell would be able to divide because there would be nothing to divide. This is the period of time where the cell grows, creates necessary proteins, and most importantly duplicates its chromosomes.

Prophase: the chromosomes condense and become rodlike and distinct chromosomes suddenly appear under the light microscope; nucleoli and nuclear envelope disappear

Metaphase: chromosomes are aligned in a plane on the metaphase plate at equator.  

Anaphase: diploid set of daughter chromosomes move toward each opposite poles;

Telophase: opposite to prophase and characterized by a cleavage furrow, which leaves the midbody (containing overlapping polar microtubules) between the newly forming two identical daughter cells; reformation of the nuclear envelope around the condensed chromosomes in the daughter cells; reappearance of nucleoli.

3. A woman with seasonal vasomotor rhinitis, who works as a train dispatcher and is an outpatient, should be prescribed an antihistaminic drug that does not suppress central nervous system. Name this drug:

Explanation

H1 receptor antagonists can be subdivided into 3 generations of drugs:

I generation antihistamine: chloropyramine, diphenhydramine, clemastine, cyproheptadine, promethazine, quifenadine and hydroxyzine.

Such drugs express a strong sedative effectare well dissolved in fats andin addition to H1- receptors, block cholinergicmuscarinic and serotonin receptors. They also easily permeate the blood-brain barrierwhich explains the sedative effect.

II generation antihistamines: loratadine, terfenadineastemizoleacrivastinedimethindenelevocabastinazelastin.

The above mentioned preparations have almost no cholinolytic effects, no sedative effects (it does not suppress CNS) and differ only by selectivitively inhibiting to H1-receptors.

Howeverthey may express cardiotoxic effectsThis group is characterized by rapid onset and prolonged clinical effectThe prolonged action time is achieved due to increased level of protein bindingcumulation of the drug and its metabolites in the bodyand slow excretion.

Indications: bronchial asthmaatopic dermatitispollen feverand allergic rhinitis.

III generation antihistamines: cetirizinefexofenadine and deslorantadineor metabolites.

These antihistamine drugs are active metabolites of I and II generation antihistamines.

4. During sanitary and bacteriological testing of water with the membrane filter technique there were revealed two red colonies on the membrane filter (Endo medium) through which 500 ml of water was filtered. Calculate the coli index and coli titer of this water:

Explanation

Coli titer is the smallest amount of water where 1 E.coli is present.

Coli index is the amount of E. coli in 1 liter of water.

            2 E.coli   -  500ml

            1 E.coli   -  ?

                        ? = 250

Therefore, Coli titer = 250

1 liter of water  =  1000ml

            2 E. coli   -  500ml

                   ?       -   1000ml (1L)

                        ? = 4

Therefore, Coli index = 4

Coliform index (Coli index) and Coli titer are used to rate the purity of water, soil and air based on the count of fecal bacteria by testing for coliforms especially the well known Escherichia coli (E. coli).

5. Protective function of saliva is based on several mechanisms, including the presence of enzyme that has bactericidal action and causes lysis of complex capsular polysaccharides of staphylococci and streptococci. Name this enzyme:

Explanation

Lysozyme, an enzyme found in tears, saliva and egg whites, digests the coating of some bacteria. The enzyme is antibacterial because it degrades the polysaccharide that is found in the cell walls of many bacteria.
6. What kind of muscle contraction occurs in an upper limb during an attempt to lift a load beyond one’s strength?

Explanation

Muscle contraction is classified into 2 types: Isotonic and Isometric.

·        Isotonic contraction: the tension in the muscle remains the same but the length of the muscle fiber is changing (Iso = same; tonic = tension) e.g. simple flexion of arm, where shortening of muscle fibers occurs but the tension does not change.

·        Isometric contraction: the length of muscle fibers remains the same but the tension is increased e.g. pulling any heavy object when the muscles become stiff and strained with increased tension but the length does not change. Holding (but not moving) – length does not change.

7. The patient’s pyramids of the medulla oblongata are damaged by tumor growth. As a result the conduction of nervous impulses will be impaired in the following pathway:

Explanation

The corticospinal tract serves as the motor pathway for upper motor neuronal signals coming from the cerebral cortex and from primitive brainstem motor nuclei. Cortical upper motor neurons originate from Brodmann areas 1, 2, 3, 4, and 6; and then descend in the posterior limb of the internal capsule, through the crus cerebri, down through the pons, and to the medullary pyramids, where about 90% of the axons cross to the contralateral side at the decussation of the pyramids of the medulla oblongata. They then descend as the lateral corticospinal tract. These axons synapse with lower motor neurons in the ventral horns of all levels of the spinal cord. The remaining 10% of axons descend on the ipsilateral side as the ventral corticospinal tract. These axons also synapse with lower motor neurons in the ventral horns. Most of them will cross to the contralateral side of the cord (via the anterior commissure) right before synapsing.
8. A patient complains of pain in the upper umbilical region. On palpation there is a mobile painful intestine. What intestine is being palpated by the doctor?

Explanation

Organs of abdominal cavity are projected on these regions of ventral abdominal wall.

The liver (right part), gallbladder, right flexure of colon, upper part of right kidney with suprarenal gland are projected on right subcostal region.

Left part of liver, stomach, upper half of duodenum, pancreas, abdominal aorta, celiac trunk and celiac plexus – epigastric region.

Cardiac part and fundus of stomach, spleen, left flexure of colon, upper part of left kidney with suprarenal gland, pancreas tail – left subcostal region.

Ascending colon, right kidney, loops of ileum intestine, right ureter – right lateral region.

Transversus colon, lower half of duodenum, large curvature of stomach, gate of kedneys, abdominal aorta and inferior cava vein. – umbilical region.

Descending colon, left kidney, loops of small intestine, left ureter – left lateral region.

Loops of small intestine, urinary bladder, lower part of ureters – pubic region.

Caecum with apendix, final part of ileum intestine, right ureter – right inguinal region.

Sigmoid colon, loops of small intestine, left ureter – left inguinal region.

9. After inoculation of investigated material (feces) on 1% alkaline peptone water and 8-hour-long incubation in the thermostat under 37oC there is growth of pale bluish film observed. Such cultural properties are characteristic of the agent of the following disease:

Explanation

The causative agents of cholera are the classical Vibrio cholera. Cholera vibrios are facultative (anaerobes). The optimum growth temperature is 37° C, and growth is arrested below 14 °C and above 42° C. The organisms grow readily on alkaline media at pH 6.0-9.0, and on solid media the colonies are transparent with a light-blue hue, forming domes with smooth edges. On gelatin the organisms produce transparent granular colonies which, when examined under a microscope, resemble broken glass. In 48 hours the medium surrounding the colonies becomes liquefied and the colonies sink into this area. Six-hour-old cultures on alkaline meat broth and peptone water produce a pellicle, which consists of cholera vibrios.
10. Histological investigation of the uterine scrape of the 45-year-old woman with disturbed ovarian menstrual cycle revealed increased number of endometrial glands, some of which are serrated, while others are dilated and cyst-like. Make the diagnosis:

Explanation

According to international classification (1982), such processes as glan­dular endometrial hyperplasia, cystic glandular endometrial hyperplasiaendometrial polyps belong to benign endometrial diseases.

Glandular endometrial hyperplasia with cellular proliferation, adeno­matous hyperplasia and adenomatous polyps are precancerous uterine diseases.

Cystic glandular hyperplasia, which is found in postmenopausal women or in reproductive period belongs to precancerous uterine lesions.

Glandular endometrial hyperplasia and cystic glandular endometrial hyper­plasia are different stages of the same process. Difference between them is presence or absence of cysts in endometrial hyperplasia (that is cystic glandular endometrial hyperplasia contains cyst; while glandular endometrial hyperplasia has no cyst). Atypical cel­lular signs at these diseases are not present. The common endometrial polyp is made up of endometrial tissue.

"Endometrial cystic glandular hyperplasia - dilated and cyst-like"

11. A pregnant woman with several miscarriages in anamnesis is prescribed a therapy that includes vitamin preparations. What vitamin facilitates carrying of a pregnancy?

Explanation

Alpha Tocopherol ( vitamin E) is a water soluble vitamin with Antioxidant function. It protects the baby by destroying reactive oxygen species.

Folic acid - Vitamin B9

Cyanocobalamin- Vitamin B12 ( megaloblastic anemia)

Pyridoxal phosphate - Vitamin B6

 
12. A patient has decreased concentration of magnesium ions that are required for ribosomes connection to granular endoplasmic reticulum. This condition is known to disturb the process of protein biosynthesis. Disturbance occurs at the following stage:

Explanation

Ribosomes on the rough endoplasmic reticulum are called 'membrane bound' and are responsible for the assembly of many proteins. This process is called translation. Magnesium ion is the divalent cation essential for the activity of several enzymes involved in the processes of translation, transcription, and replication. Mg2+ ions play a vital role in the activity of enzymes such as DNA polymerase, RNA polymerase, RNase, Proteases, etc.

The role of Mg2+ ions are:

  1. To regulate the association and dissociation of the ribosomal subunits.
  2. To bind to the Mg2+ binding sites of the ribosomal proteins and regulate the series of conformation changes in the ribosome required during the translation.
  3. The concentration of the Mg2+ ions also determine the final protein product.
13. A patient with chronic heart failure presents with increased blood viscosity. Capillaroscopy detected damage to the vessel walls of the microcirculation system. What disorder is possible in the given case?

Explanation

Sludge phenomenon is a microscopic process that occurs in blood vessels. Occurs due to increased viscosity in blood and tendency of RBC to aggregate. A thrombus is a blood clot that adheres to blood vessels. Emboli are thrombi,tissues or foreign particles  that move away from the area of production and travel through the bloodstream.

 
14. A 3-year-old boy with pronounced hemorrhagic syndrome has no antihemophilic globulin A (factor VIII) in the blood plasma. Hemostasis has been impaired at the following stage:

Explanation

Anti-hemophilic globulin (AHIG, Platelet cofactor-I. Anti-hemophilic factor A (AHF-A). This is the compound called factor VIII. Deficiency of this factor results in Hemophilia A.

There are two systems of clotting, intrinsic and extrinsic, which converge upon what is called the final common pathway.

Intrinsic system (Internal mechanism of prothrombinase activation) This system is so called, because all factors taking part in the process are derived from the blood itself and it can take place in pure blood (blood not contaminated with tissue juice) kept in a test tube. This process takes place in the following six stages:

Each of these steps is regulated by plasma and cellular co-factors and Ca2+. The inactive and active blood clotting factors are distinguished by writing and 'a' respectively after the factor.

Stage 1: Three plasma proteins, i.e. Hageman factor (XII), high molecular weight kininogen and pre-kallikrein form a complex with vascular subendolhelial collagen. 

Satge 2: Factor XII-a converts factor XI-i to XI-a.

Stage 3: Factor XI-a converts factor IX-i to IX-a.

Stage 4: Factor IX-a in the presence of factor VIII, Ca2+ a platelet membrane lipoprotein (platelet factor 3) converts X to X-a.

Stage 5: Several factors take part in the conversion of prothrombin to thrombin. These include factor X-a, factor V-a, Ca2+ and phospholipids. 

Stage 6: Conversion of fibrinogen to fibrin is brought about by thrombin 

15. Paronychia of the patient’s little finger was complicated with phlegmon of the hand and forearm. In this case the suppuration had spread through the:

Explanation

  In the image above you can clearly see that the Vagina synovialis communis mm. flexorum (common flexor sheath) is continuous with that of the fifth finger. Hence, any infection of the synovial tendinous sheath of the fifth finger can easily spread through the Vagina synovialis communis mm. flexorum to the hand and forearm.
16. During cholecystectomy besides a. cystyca another artery was pulled into the ligature. Ligation of this artery resulted in right-sided necrosis of the liver which led to the death of the patient. What artery was mistakenly ligated along with a. cystyca?

Explanation

  Anatomically, the cystic artery (a. cystica), which supplies the gall bladder and the right branch of hepatic artery proper (Ramus dexter a. hepatica propria) are very close. The right hepatic artery supplies the right side of the liver. Accidental ligation of these two arteries during cholecystectomy is a common occurrence among inexperienced surgeons.
17. During treatment with bismuth preparations a patient with syphilis developed gray spots on his oral mucosa and nephropathy symptoms. What drug is used as an antidote to bismuth preparations poisoning?

Explanation

Dimercaprol (unithiol, British anti-lewisite) is used to chelate mercury, bismuth, arsenic and in combination with edentate calcium disodium to treat lead intoxication. It contains two sulfuhydryl (SH-) groups and forms two bonds with metal ions. Administered intramuscularly.

Nalorphine hydrochloride – antidote, blocks opoid receptors and replace opoids from binding with them.

Bemegride - an analeptic used to treat acute barbiturate overdose.

Naloxone - is indicated for known or suspected opioid overdose.

Methylene blue - it is mainly used to treat methemoglobinemia; mild antiseptic to kill bacteria

18. A patient with periodontitis of the lower molar came to the doctor. It was determined that the inflammatory process spread to the lymph nodes. What lymph nodes were the first to be affected by the inflammatory process?

Explanation

[caption id="attachment_1081" align="aligncenter" width="1024"] Supraclavicular Lymph Nodes Diagram Anatomy Of Cervical Lymph Nodes - Human Anatomy - Anatomy Organ[/caption] The submandibular lymph nodes sit between the submandibular salivary glands, which are underneath the tongue, and the mandible, or lower jawbone. Teeth (lower molar teeth) drain into the submandibular and deep cervical lymph nodes. Anterior part of mouth floor drain into submental and upper deep cervical while posterior part into submandibular and upper deep cervical.
19. A 63-year-old man suffers from esophageal carcinoma, presents with metastases into the mediastinal lymph nodes and cancerous cachexia. What pathogenetic stage of neoplastic process is observed in the patient?

Explanation

Neoplastic growth: the main three stages are - transformation, promotion and progression.

Transformation. The first stage; it is followed with the cell oncogene activation. The cell acquires unusual property, which is called immortalisation. This is a potential unlimited division, immortality ability. 

Promotion. Supplementary influences upon immortalisated cell, are necessary to exit it out of the latent state, for giving a push to irrepressible division. These are provoking factors, which are supplementary doses of chemical cancerogenes or x-rays, retroviral superinfection. They are named promotors.

Progression is the very last and the most protracted stage of neoplastic growth development. The final progression stage of any tumor is its transformation into the malignant neoplasm. The major criteria of malignant tumor is its ability to generalisation, that is – to metastasing. Metastasing includes three stage: neoplastic invasion into the surrounding tissues, tumor cells transport with the blood and lymphatic vessles (metastases into the mediastinal lymph nodes), their implantation indifferent organs and tissues.

20. A person is in a room with air temperature of 38oC and relative air humidity of 50%. What type of heat transfer ensures maintenance of constant body core temperature under these conditions?

Explanation

Evaporation is a way the body dissipates heat to the environment by its evaporation via sweat or evaporation of moisture from the skin and respiratory tract mucous membranes of (“wet” heat loss). Evaporation closely related to relative humidity.

Heat Radiation is a way the surface of the human body emits heat to the environment in the form of infrared rays. The amount of heat the body radiates to the environment is proportional to the surface of radiation area and to the difference between the mean values of skin and environment temperature. The surface radiation area is the total surface area of body parts that contact the air. Elimination of heat by radiation increases with a decrease in ambient temperature and decreases with its increase. It is possible to reduce elimination of heat by radiation via reduction of the surface of radiation area (“winding oneself into a ball”). Heat radiation does not require a medium for transfer of heat. (Key words: naked or lightly clothed).

Convection is a way the body eliminates heat by means of transferring heat via moving particles of air or water. To dissipate heat by means of convection, body surface shall be airflowed at a temperature that is lower than the temperature of the skin. At that, air layer contacting with the skin warms up, decreases its density, rises and is replaced by cooler, denser air. By increasing the speed of the air flow (wind, ventilation) heat emission increases significantly as well (forced convection). Convection requires convection current; current of gases or liquids (Key words: air over exposed area of skin).

Conduction is a way the body eliminates heat by means of direct contact with another object. Heat is transferred down the temperature gradient (i.e. from the object of higher temperature to the object of lower temperature). Conduction requires contact with another object (Key words:  in water).

21. A patient has hoarseness of voice. During laryngoscopy a gray-white larynx tumor with papillary surface has been detected. Microscopic investigation has shown the following: growth of connective tissue covered with multilayer, strongly keratinized pavement epithelium, no cellular atypia. What is the most likely diagnosis?

Explanation

Papillomas are benign epithelial neoplasms producing microscopically or macroscopically visible fingerlike or warty projections from epithelial surfaces. Polyp: when a neoplasm – benign or malignant produces a macroscopically visible projection above a mucosal surface and projects, for example, into the gastric or colonic lumen. Fibroma: benign, node of differentiated connective tissue. Angioma and angiofibroma are related to vessels.
22. A shepherd, who tended to the flock of sheep with his dogs, gradually developed pain in the chest and bloody expectorations. X-ray revealed spheric helminth larvae in the patient’s lungs. Specify the helminth that could be the causative agent of this disease:

Explanation

·        Echinococcus granulosus causes Echinococcosis. It is composed of a scolex and only 3 proglottids, making it one of the smallest tapeworms. The scolex has a circle of hooks and 4 suckers similar to Taenia solium. Dogs are the most important definitive hosts. The intermediate hosts are usually sheep. Humans are almost always dead-end intermediate hosts. Transmission: ingestion of eggs from dog faeces. Disease – hydatid cysts in liver causing anaphylaxis if antigens released.

·        Diphyllobothrium latum causes diphyllobothriasis. A fish tapeworm. In contrast to the other cestodes, which have suckers , the scolex of D. latum has two elongated sucking grooves by which the worm attaches to the intestinal wall. Infection by D. latum causes little damage in the small intestine. In some individuals, megaloblastic anemia occurs as a result of vitamin B12 deficiency caused by preferential uptake of the vitamin by the worm. Transmission: ingestion of larvae from raw freshwater fish. Caviar is prepared from fish.

·        Taeniasis: there are two important human pathogens in the genus Taenia: T. solium (pork tapeworm) and T. saginata (beef tapeworm)

·        Trichiniasis (Trichinosis) is caused by Trichinella spiralis (nematode- roundworm). Transmission: fecal-oral; undercooked meat (especially pork). A few days after eating undercooked meat, usually pork, the patient experiences diarrhea followed by 1-2weeks later by fever, muscle pain, periorbital edema and eosinophilia.

·        Ascaridiasis (Ascariasis) caused by Ascaris lumbricoides (giant roundworm). The major damage occurs during larval immigration rather than from the presence of the adult worm in the intestines. The principal sites of tissue reaction are the lungs, where inflammation with an eosinophilic exudates occurs in response to larval antigens. Ascaris pneumonia with fever, cough and eosinophilia can occur with a heavy larval burden.

23. A 67-year-old patient with clinical diagnosis of chronic bronchitis, pneumosclerosis, and cardiopulmonary decompensation has the biopsy material taken from the suspicious area in his right bronchus mucosa. Cellular and tissue atypism along with pearly bodies can be histologically detected. What pathologic process is characterized by the described histological changes?

Explanation

Squamous cell (epidermoid) carcinoma: these tumors usually arise in a large bronchus and are prone to massive necrosis and cavitation. The tumor is  diagnosed microscopically by identification of either intercellular bridges or keratinization. Usually the spread of squamous cell carcinoma is more rapid than the other histologic types. Frequently, the edge of the growth and the adjoining uninvolved bronchi show squamous metaplasia, epithelia dysplasia and carcinoma in situ.
24. A child with point mutation presents with absence of glucose 6-phosphatase, hypoglycemia, and hepatomegaly. What pathology are these signs characteristic of?

Explanation

Glycogen storage diseases: 12 types, all resulting in abnormal glycogen metabolism and an accumulation of glycogen within cells. Type I (Von Gierke disease): findings – severe fasting hypoglycemia, ↑↑glycogen in liver, ↑blood lactate, ↑triglycerides, ↑uric acid and hepatomegaly. Deficient enzyme is glucose-6-phosphatase. It is autosomal recessive.

Deficiency of amyloid-1,6-glycosidase (Type III-Cori’s disease); deficiency of glycogen phosphorylase (Type V-McArdle’s; Type VI-Hers’)
25. At the 2-3 day after the gastric resection the patient’s intestinal peristalsis failed to restore. What should the patient be prescribed to stimulate the function of his gastrointestinal tract?

Explanation

Proserin (neostigmine) is an indirect acting cholinergic agonist (anticholinesterase). It preserves endogenous acetylcholine which can stimulate a greater number of acetylcholine receptors at the muscle endplate. Thereby increasing intestinal peristalsis and tone of urinary bladder.
26. Examination of the patient with traumatic brain injury revealed that he has lost the ability to discern the movement of an object on the skin. What part of the cerebral cortex is damaged?

Explanation

The post central gyrus is responsible for sensation

The pre central gyrus- for motor activities.

The calcarine sulcus( occipital lobe)- for sight

The Broca’s area ( frontal lobe)- for speech articulation.

 
27. Blood test of the patient revealed albumine content of 20 g/l and increased activity of lactate dehydrogenase isoenzyme 5 (LDH5). These results indicate disorder of the following organ:

Explanation

Albumin is a protein specifically synthesized in the liver. Normally ranges from 35-55g/L. Low albumin level, elevated  Lactate dehydrogenase 5 (LDH5), elevated alkaline phosphatase are markers for liver disorders.Increased Aspartate and alanine transaminase levels (ALT &AST) are also used but not specific because they increase in disorders of some other organs eg, heart and kidney. Creatine phosphokinase (MB isoform) is a marker for heart related issues.

 
28. Section shows significant enlargement of the patient’s right kidney. There is a nephrolith at the place of incision. Renal pelvic lumen is distended with accumulating urine. Renal parenchyma is acutely thinned out. What is the most correct diagnosis?

Explanation

 Hydronephrosis refers to the dilation of renal pelvis and calyces due to partial or intermittent obstruction of the outflow of urine.

 
29. A patient demonstrates sharp decrease of pulmonary surfactant activity. This condition can result in:

Explanation

 

Pulmonary surfactant is a lipoprotein produced by type II alveolocytes and secreted into the alveolar space. It is responsible for the reduction of surface tension between air/liquid thereby preventing the lungs from collapse.

30. After a case of cold the patient developed a lacrimation disorder. This disorder was caused by functional disturbance of the following autonomic ganglion:

Explanation

 

The pterygopalatine ganglion or sphenopalatine ganglion is a parasympathetic ganglion that supplies the lacrimal gland and nasal mucosa. The Otic ganglion supplies the posterior ⅓ of the tongue and the parotid gland for salivation. The ciliary ganglion supplies the ciliary and sphincter pupillae muscles of the eye.

31. A patient is diagnosed with diabetic coma. Blood sugar is 18,44 mmol/l. What glucose-regulating drug should be prescribed in the given case?

Explanation

 

Insulin is a hormone produced by the beta cells present in the islets of langerhans in the pancreas. It stimulates tissue uptake of glucose, thereby reducing blood glucose level. A person in a diabetic comatose state should immediately be administered rapid acting insulin eg,Humalog.

32. A man came into the admission room with complaints of edemas, rapid heart rate, dyspnea, and cyanotic mucosal tunics. He was diagnosed with chronic heart failure. What drug should be prescribed to improve the patient’s general state?

Explanation

Cardiac insufficiency: Cardiac glycoside is the most appropriate in this case. Cardiac glycosides are glycosidic drugs of plant origin having cardiac inotropic property, increase myocardial contractility and output.

Digitoxin is long-acting but basically administered orally and used mainly for chronic heart failure (also digoxin).

33. Exposure to colchicine resulted in metaphase plate of a human containing 23 chromosomes more than it is normal. Name this mutation:

Explanation

When separation does not occur, it is called non-disjunction and both members of a pair of homologous chromosome move into one cell. In mitotic non-disjunction, mosaicism is produced with some cells having an abnormal chromosome number and others being normal; an extra chromosome is present (trisomy – polysomy - polyploidy) or one is missing (monosomy). Polyploidy occurs when the entire chromosome set is present in more than two copies, so the individual may be triploid rather than diploid and have a chromosome number of 69.
34. A 30-year-old man complains of suffocation, heaviness in the chest on the right, general weakness. Body temperature is 38,9oC. Objectively the right side of the chest lags behind the left side during respiration. Pleurocentesis yielded exudate. What is the leading factor of exudation in the patient?

Explanation

 Pleurocentesis is a procedure used in removal of fluid or air from the pleural space.These exudates accumulate due to increased permeability of blood vessels.

 
35. A 46-year-old woman suffering from cholelithiasis developed jaundice. Her urine became dark yellow, while feces are lightcolored. What substance will be the most increased in concentration in the blood serum in this case?

Explanation

 

Obturation (obstruction, to close) of bile duct – it can be:

* Intrahepatic – blockage of intrahepatic bile ducts

* Extrahepatic – blockage of common bile duct (ductus choledochus).

Findings:

* malabsorption: bile salts do not enter the Small Intestine; no emulsification of fat.

*light coloured stool: due to lack of urobilin (which leads to lack of stercobilin).

*Jaundice (posthepatic, mechanic, obstructive): increased conjugated Bilirubin.

Steatorrhea

The findings are specific for obstruction of bile duct and bile acid deficiency.

Cholelithiasis is obstruction of bile duct by a stone.

Indirect; Hemolytic; Prehepatic

Mixed; Parenchymal; Hepatic

Direct; Obstructive; Mechanic; Posthepatic

Stercobilin (faeces)

        ↑↑↑

Decreases (pale faces)

Absent (clay coloured faeces)

Type of bilirubin in blood

Unconjugated

Conjugated and Unconjugated

Conjugated

36. A traumatology unit received a patient with crushed muscular tissue. What biochemical indicator of urine will be raised in this case?

Explanation

Creatine kinase/Creatine phosphokinase is an enzyme present in striated muscles, used to generate energy. When its serum level is elevated it is often an indication of muscle damage caused by injury, muscular dystrophy or cardiac problems. Most forms of muscular dystrophy are associated with decreased creatinine excretion. Creatinine is a break down product of creatine. Normal constituent of urine but can be elevated in muscular dystrophy.
37. A 30-year-old woman first developed pain, swelling, and skin redness in the area of joints about a year ago. Provisional diagnosis is rheumatoid arthritis. One of the likely causes of this disease is change in the structure of the following connective tissue protein:

Explanation

    Rheumatoid (atrophic) arthritis is a chronic progressive inflammatory arthritis of unknown origin involving multiple joints and characterized by disorganization of connective tissue (collagen) of the synovial membrane and articular cartilage and development of their deformation. Disorganization of the connective tissue cause increase in proteoglycans and glycosaminoglycans (GAGs) concentration in blood. Proteoglycans and GAGs are responsible for the physical properties of ground substance. Main morphological appearance of rheumatoid arthritis is synovitis. It has 3 stages: in the first stage, it is characterized by an acute inflammatory reaction with development of edema, hyperemia and infiltration of lymphocytes, plasma cells and macrophages. Small areas of superficial necrosis or superficial erosions are covered by fibrinoid deposits. Not infrequently 2-3mm “rice bodies” (rice grains) composed of fibrin, fibronectin, collagen and immunoglobulin are present in joint cavities of seropositive patients.

Collagen is a major component of the connective tissue.

38. A 15-year-old teenager complains of lack of air, general weakness, palpitations. Heart rate is 130/min., BP is 100/60 mm Hg. ECG: QRS complex has normal shape and duration. The number of P waves and ventricular complexes is equal, T wave merges with P wave. What type of cardiac arrhythmia is observed in the teenager?

Explanation

Sinus tachycardia is the increase in discharge of impulses from the sinoatrial (SA) node, resulting in increase in heart rate (heart rate increase up to 100beats/min).ECG is normal, except for short R-R interval (indicated by the merging of T waves with P waves).

Sinus bradycardia is the reduction in discharge of impulses from SA node resulting in decrease in heart rate. Heart rate is less than 60beats/min. Extrasystole is the premature contraction of the heart before its normal contraction. ECG is altered. Sinus arrhythmia is characterized by irregular generation of impulses and may be due to variations in the tone of the vagus nerve. ECG is altered.

39. A patient complaining of dizziness, thirst, difficult swallowing, and impaired vision of close objects has addressed a doctor. Objectively: respiratory rate is increased, pupils are dilated, general agitation, talkativeness, though the speech is indistinct. BP is 110/70 mm Hg, heart rate is 110/min. Given symptoms can indicate overdosage of the following drug:

Explanation

 Atropine is an antimuscarinic agent that causes mydriasis ( pupil dilation), increases salivation, heart rate and breathing rate.

Aminazine is an antipsychotic drug. Morphine is an opioid analgesic used in pain syndrome.

 
40. On autopsy of a 40-year-old woman, who had been suffering from rheumatoid arthritis, her liver is found to be dense and enlarged. On dissection its tissue is red-brown colored, with enlarged follicles resembling semi-transparent grayish-white granules. What is the most likely pathological process?

Explanation

Amyloidosis of Spleen are observed in two patterns;

Sago Spleen- possess translucent  pale and waxy nodules resembling sago granules (  semi-transparent greyish white).

Lardaceous spleen- possess map-like areas of amyloid deposits.

 
41. On autopsy the dissector determined that the lungs are enlarged, pale, soft, do not deflate, crunch when cut. Microscopically there are dilated alveolar ducts, alveolar septa are thin, and signs of intracapillary sclerosis are observed. What pulmonary disorder are these presentations characteristic of?

Explanation

Emphysema is a condition of the lung characterised by permanent dilation of the air spaces distal to the terminal bronchioles, accompanied by the destruction of their walls without obvious fibrosis.

 

Pneumonia is simply an acute inflammation of the respiratory tract. Phases- congestion and edema, red and gray hepatization and resolution phase.

42. During removal of the hyperplastic thyroid gland of a 47-year-old woman, the parathyroid gland was damaged. One month after the surgery the patient developed signs of hypoparathyroidism: frequent convulsions, hyperreflexia, laryngospasm. What is the most likely cause of the patient’s condition?

Explanation

 

Parathyroid glands are responsible for secretion of parathormone which increase and maintain blood level calcium, stimulates osteoclast activities. Has the opposite function of calcitonin which stimulates bone resorption. A decrease in parathormone due to parathyroid gland damage leads to hypocalcemia.

43. On examination the patient presents with hirsutism, moon-shaped face, stretch marks on the abdomen. BP is 190/100 mm Hg, blood glucose is 17,6 mmol/l. What pathology is such clinical presentation characteristic of?

Explanation

Cushing Syndrome: Etiology *INCREASE Cortisol due to a variety of causes (Glucocorticoids). *Exogenous corticosteroids:  result in DECREASE ACTH (most common cause - MCC). *Primary adrenal adenoma, hyperplasia or carcinoma (Cushing’s Syndrome). *ACTH-secreting pituitary adenoma (Cushing Disease). Findings: Hypertension, Weight Gain, Moon Facies, Truncal Obesity, Buffalo Hump, Skin Changes (thinning striae), Osteoporosis, Hyperglycemia (Insulin resistance), Amenorrhea, Immunosuppression.
44. Histological specimen of the ovary shows large hollow structures. Primary oocyte within these structures is surrounded with transparent membrane and radiating crown and is situated in the cumulus oophorus, the wall is made of follicular cell layer and theca. What ovarian structure can be characterized by these morphological features?

Explanation

The Primary follicle presents with 1-2 layers of cells; secondary follicles have more cells with accumulation of follicular fluid.Mature/tertiary/graafian follicles are surrounded by granulosa cells called the cumulus oophorus, theca cells form their basement membrane.

 
45. Histological specimen demonstrates a parenchymal organ with cortical and medullary substances. The cortical substance is composed of bands of epithelial cells with capillary blood vessels between them. The bands form three zones. The medullary substance consists of сhromaffin cells and venous sinusoids. What organ can be characterized by these morphological features?

Explanation

The adrenal gland is made up of the cortex and medulla.The adrenal cortex is made of 3 zones namely;

Zona Glomerulosa: Mineralocorticoids eg. aldosterone

Zona Fasciculata; Glucocorticoids

Zona Reticularis: Androgens.

The medulla consists of chromaffin responsible for the secretion of adrenaline and noradrenaline.

 
46. A child with suspected colienteritis was delivered to the infectious diseases hospital. Colibacillus was obtained from the child’s feces. How to determine whether this bacillus is of pathogenic variety?

Explanation

 Agglutination reaction is a method of diagnosis carried out with an antigen (from sample collected from patient) and a serum. Formation of clumps confirms the diagnosis.

 
47. A patient ungergoes right-sided pulmonectomy due to lung cancer. Name the anatomical structures of the right lung radix (downward order):

Explanation

48. In a township there was registered an outbreak of hepatitis, which was attributed to water supply. What hepatitis virus could be the cause of the outbreak in this township?

Explanation

Hepatitis E: incubation period 15 - 45days; it is an RNA-virus; transmission is fecal-oral especially waterborne. It is not transmitted by blood or sexually. It mostly occurs in developing countries. It produces acute hepatitis and fulminant hepatitis may develop in pregnant women.
49. A 64-year-old woman presents with disturbed fine motor function of her fingers, marked muscle rigidity, and tremor. The neurologist diagnosed her with Parkinson’s disease. What brain structures are damaged resulting in this disease?

Explanation

Parkinson disease is associated with a loss of dopaminergic neurons in the substantia nigra. Parkinsonism is a progressive neurological disorder of muscle movement, characterized by tremors, muscular rigidity, bradykinesia (slowness in initiating and carrying out voluntary movements) and postural and gait abnormalities. Most cases involve people over the age of 65.

The disease is correlated with destruction of dopaminergic neurons in the substantial nigra with a consequent reduction of dopamine actions in the corpus striatum, parts of the brain’s basal ganglia system that are involved in motor control.

The substantial nigra, part of the extrapyramidal system is the source of dopaminergic neurons that terminate in the neostriatum.

Treatment: drugs – levodopa, carbidopa, selegiline, rasagiline

So far, levodopa has been the only drug tested on Parkinson.

50. Due to prolonged taking of a drug the patient can develop osteoporosis, gastric mucosal erosions, hypokalemia, sodium and water retention, and decreased blood content of corticotropine. Specify this drug:

Explanation

Osteoporosis is a common adverse effect of long-term corticosteroid therapy, due to the ability of glucocorticoids (prednisolone) to suppress intestinal Ca2+ absorption, inhibit bone formation and decrease sex hormone synthesis. Other adverse effects include peptic ulcer (erosions of stomach mucosa), hypokalemia, emotional disturbances and the classic cushing-like syndrome (i.e. redistribution of body fat, puffy face, increased body hair growth, acne, insomnia and increased appetite) are observed when excess corticosteroids are present. Excess corticosteroid results in decreased corticotrophin (adrenocorticotropic hormone) through the negative feedback mechanism.
51. Nitrogen is being excreted from the body mainly as urea. When activity of a certain enzyme in the liver is low, it results in inhibition of urea synthesis and nitrogen accumulation in blood and tissues. Name this enzyme:

Explanation

  Urea is the major disposal form of amino groups derived from amino acids and account for about 90% of the nitrogen-containing components of urine. Formation of carbomoyl phosphate by carbomoyl phosphate synthetase I is driven by cleavage of two molecules of ATP. Ammonia is incorporated into carbamoyl phosphate. Ultimately, the nitrogen atom derived from this ammonia becomes one of the nitrogens of urea. NB: carbamoyl phosphate synthetase II participates in the biosynthesis of pyrimidines. However, when the function of carbomoyl phosphate synthetase I is compromised, blood ammonia levels rise and urea synthesis is disturbed.
52. After pancreatic surgery the patient developed hemorrhagic syndrome with disturbed 3rd stage of blood clotting. What will be the most likely mechanism of the hemostatic disorder?

Explanation

Blood Hemostasis includes 3 phases; 

I-  Vascular spasm ( vasoconstriction)

II- Plug formation ( intrinsic/extrinsic pathway, activation of thrombin and activation of fibrinogen)

III- plug retraction ( fibrinolysis)

 
53. A patient with jaundice has high total bilirubin that is mainly indirect (unconjugated), high concentration of stercobilin in the feces and urine. The level of direct (conjugated) bilirubin in the blood plasma is normal. What type of jaundice can be suspected?

Explanation

Hemolytic/Prehepatic Jaundice is the type of Jaundice that occurs because of excessive destruction of RBCs resulting in ↑ blood level of free, Indirect, unconjugated bilirubin. In this condition, the excretory function of the liver is normal. But the quantity of bilirubin ↑ enormously. The liver cells cannot excrete that much excess bilirubin rapidly. Formation of Urobilinogen (stercobilin) ↑ resulting in the excretion of more amount of stercobilin in stool and urine.

Indirect; Hemolytic; Prehepatic

Mixed; Parenchymal; Hepatic

Direct; Obstructive; Mechanic; Posthepatic

Stercobilin (faeces)

↑↑↑

Decreases (pale faces)

Absent (clay coloured faeces)

Type of bilirubin in blood

Unconjugated

Conjugated and Unconjugated

Conjugated

  • Gilbert syndrome (familial nonhemolytic Jaundice): Autosomal recessive defect. Impaired UGT activity. Jaundice occurs with fasting, volume depletion, stress, menses.

  • Physiologic Jaundice of newborn: begins on day 3 of life. Caused by normal macrophage destruction of fetal RBCs containing HbF and inability of the newborn’s liver to handle excess load.

54. Histological specimen shows organ parenchyma to consist of lymphoid tissue that forms lymph nodules; the nodules are located diffusely and have a central artery. What anatomical structure has such morphological characteristics?

Explanation

    The spleen is about the size of a clenched fist and is the largest lymphatic organ. It is located in the upper left quadrant of the abdominal cavity and has a rich blood supply. Most of the spleen consists of splenic pulp. Splenic pulp is divided into two regions: white and red pulp. White pulp consists of a thick accumulation of lymphocytes surrounding a central artery. Lymphocytes that aggregate around the central artery constitute the periarterial lymphatic sheath (PALS). The red pulp contains large numbers of RBCs that it filters and degrades.
55. During fibergastroscopy of a patient with ulcer disease of the stomach, the mucosal biopsy material is taken from the area of an ulcer. Impression smear is prepared from the biopsy material and stained by Gram method; the rest of the biopsy material is tested for urease activity. Microscopy of the impression smear revealed gram-negative spiral-shaped microorganisms, urease activity test is positive. What bacteria were detected?

Explanation

Helicobacter pylori causes gastritis and peptic ulcers. Infection with H. pylori is a risk factor for gastric carcinoma and is linked to mucosal-associated lymphoid tissue (MALT) lymphoma. It is a gram negative, flexibacteria, oxidase positive, microaerophilic, motile and the only species in the Helicobacter genus to have multiple unipolar – sheathed flagella. Microaerophiles need O2 because they cannot ferment or respire anaerobically. However, they are poisoned by high concentrations of O2. They gather in the upper part of the test tube but not the very top. CO2 is essential for initial growth of H. pylori in liquid media (microaerophilic property). They produce urease, so it’s a diagnostic tool and not a necessary consideration for cultivation (culture medium).
56. A patient suffering from gout was prescribed allopurinol. What pharmacological property of allopurinol provides therapeutic effect in this case?

Explanation

The end product of the purine nucleotides catabolism in humans and other primates is uric acid (urate) which is excreted in urine. Allopurinol and febuxostat inhibits Xanthine oxidase (XO). Hypoxanthine and Xanthine which is more soluble is excreted in urine. Purine nucleotides (adenine and guanine).  AMP – Adenosine monophosphate; GMP – Guanosine monophosphate

Pyrimidine (thymine, Uracil, cytosine); catabolism of thymine and uracil gives urea, while catabolism of cytosine gives β-alanine.

Allopurinol is a purine analog. It reduces the production of uric acid by competitively inhibiting the last two steps in uric acid biosynthesis that are catalyzed by xanthine oxidase. NOTE: Uric acid is less water soluble than its precursors. When xanthine oxidase is inhibited, the circulating purine derivatives (xanthine & hypoxanthine) are more soluble and therefore are less likely to precipitate.
57. A woman, who has been suffering from marked hypertension for 15 years, has lately developed dyspnea, palpitations, slightly decreased systolic pressure, while diastolic pressure remains the same. What is the main mechanism of heart failure development in this case?

Explanation

 

Hypertension is a condition caused by an increased vascular resistance in the walls of blood vessels. A reduced systolic pressure leads to the heart’s inability to push enough blood through this resistance leading to accumulation of a high amount of blood left in the heart even after systole (a cardiac overload).

58. Old burial ground for animal refuse, which has not been in use for the last 50 years, is planned to be given for housing development. However, the soil analysis detected viable spores of a causative agent of an extremely dangerous disease. What microorganism is the most likely to remain in the soil for such a long period of time?

Explanation

There are two medically important Bacillus species: Bacillus anthracis and Bacillus cereus. Bacillus anthracis causes anthrax. Human disease occurs in 3 main forms: cutaneous, pulmonary (inhalation) and gastrointestinal. Humans are most often infected cutaneously at the time of trauma to the skin, which allows the spores on animal products such as hides, bristles and wool to enter. Spores can also be inhaled or when contaminated meat is ingested. After being inhaled, the organism moves rapidly to the mediastinal lymph nodes and causes hemorrhagic mediastinitis. Pathogenesis is based on the production of two exotoxins  (Anthrax toxin) – edema factor and lethal factor. Hemorrhagic mediastinitis, septic shock hemorrhagic meningitis and death are severe life-threatening complications. In fatal cases, the organism may affect the spleen, liver, intestines, kidneys, adrenal glands and meninges.
59. Blood of the patients with diabetes mellitus shows increased content of free fatty acids. Name the most likely cause of this:

Explanation

 

Diabetes mellitus is a disease characterised by high levels of glucose in the blood . Tissue uptake of glucose is impaired this leads the body to search for an alternative source of energy, increasing the breakdown of other metabolites such as Fats. Triglyceride lipase is an enzyme that catalyzes the hydrolysis of fats.

60. During autopsy of a man, who had been suffering from mitral stenosis, the lungs are revealed to be dense and brown-colored. What pathologic process had occurred in the lungs?

Explanation

Hemosiderosis is characterised by breakdown of RBC. In the lungs, hemosiderin-laden macrophages or siderophages are referred to as ‘heart failure cells’.

Hemochromatosis is a genetic disorder characterised by increased absorption of iron.These iron are deposited in organs like pancreas, liver, skin, heart muscle and mesenteric lymph nodes.

 
61. Examination of the coronary arteries revealed atherosclerotic plaques with calcinosis that close the arterial opening by 1/3. In the muscle there are numerous whitish layers of connective tissue. Name the process detected in the myocardium:

Explanation

Diffuse cardiosclerosis

Diffuse – multiple whitish layers of connective tissue

Cardiosclerosis – Atherosclerotic calcified plaques

Focal cardiosclerosis. Focal –white fibrous depressed area 3cm in diameter.    Cardiosclerosis - Atherosclerosis.
62. T-lymphocytes are determined to be affected with HIV. In this case viral enzyme reverse transcriptase (RNA-dependent DNA-polymerase) catalyzes the synthesis of:

Explanation

RNA dependent DNA polymerase

     DNA polymerase – synthesizes DNA

     RNA dependent – from mRNA (on the matrix of virus mRNA)

       Human immunodeficiency virus (HIV): diploid genome (2 molecules of RNA). The 3 structural genes(i.e. proteins coded for by the genes) are:

·        env (gp 120 and gp 41): formed from cleavage of gp 160 to form envelope glycoproteins. gp 120 is for attachment to host CD4+ T cell. gp 41 is for fusion and entry.

·        Gag (p24): capsid protein

·        pol: reverse transcriptase, aspartate protease, integrase.

ELISA/Western blot (immunoblot) tests look for antibodies to the viral proteins listed above.

Reverse transcriptase synthesizes dsDNA (ds-double stranded) from genomic RNA (mRNA); dsDNA integrates into host genome. Virus binds CD4 as well as a coreceptor, either CCR5 on macrophages (early infection) or CXCR4 on I cells (late infection).

·        Homozygous CCR5 mutation – immunity

·        Heterozygous CCR5 mutation – slower course.

63. A woman with dense area in her mammary gland came to a surgeon. To minimize the trauma to the mammary gland lobule during the operation the surgeon should make the incision:

Explanation

Mammary gland: The nipple (papilla mammae) is situated on the anterior surface of the gland. It is surrounded by the areola (areola mammae). The areola contains the areolar (Montgomery’s) glands and sebaceous glands. The mammary gland is composed of acini, which make up the lobules, whose aggregation will form the lobes of the gland. The lobes are arranged in a radiating fashion and drained into lactiferous ducts. 18-20 lobes drain through 18-20 ducts, which converge or open on the nipple.

The lobes are partly separated from each other by irregular and incomplete fibrous septa, which are continuous with the gland stroma. The organ is anchored to the overlying skin and the underlying pectoral fascia by bands of fibrous tissue called the Cooper’s ligaments.

64. Electrical activity of neurons is being measured. They fire prior to and at the beginning of inhalation. Where are these neurons situated?

Explanation

Respiratory centers are group of neurons which control the rate, rhythm and force of respiration. Depending upon the situation in brainstem, the respiratory centers are classified into two groups:

·         Medullary centers consisting of:       * dorsal respiratory group of neurons

                                                                       *ventral respiratory group of neurons

·         Pontine centers:           * apneustic center

                                                 *pneumotaxic center

Dorsal respiratory group of neurons are diffusely situated in the nucleus of tractus solitaries which is present in the upper part of the medulla oblongata. Usually these neurons are collectively called inspiratory center. All neurons of the dorsal respiratory group are inspiratory neurons and generate inspiratory ramp by the virtue of their autorhythmic property. They are responsible for the basic rhythm of respiration.

Ventral respiratory group of neurons are present in nucleus ambiguous and nucleus retroambiguous in the medulla oblongata, anterior and lateral to the nucleus of tractus solitaries. Ventral respiratory group has both inspiratory and expiratory neurons.

65. Investigation of an isolated cardiac myocyte determined that it does not generate excitation impulses automatically, which means this cardiac myocyte was obtained from the following cardiac structure:

Explanation

Sinoatrial node, Atrioventricular node, His’ bundle, purkinje fibers are all parts of the conductive system of the heart and are capable of generating excitation impulses automatically. The ventricles are not capable of generating excitation impulses they only contract in response to impulses. The ventricular myocardium will only generate impulses in pathological conditions in the presence of an ectopic foci.
66. To lose some weight a woman has been limiting the amount of products in her diet. 3 months later she developed edemas and her diuresis increased. What dietary component deficiency is the cause of this?

Explanation

 Edemas are caused by i) Increase in hydrostatic pressure 

                              ii) decrease in oncotic  pressure ( maintained mainly by Albumins)

                              iii) change in osmotic pressure

Albumins are the major proteins that maintain oncotic pressure.

 
67. The patients with organic brain disorder can take the following drug to improve their memory:

Explanation

Piracetam is a nootropic drug. It improves brain function eg. memory. Diazepam,nitrazepam and medazepam are tranquilizers used in the treatment of anxiety.

 
68. A 40-year-old man developed skin redness and swelling in the neck area, where eventually a small abscess appeared. On section the focus is dense and yellowgreen colored. In the purulent masses there are white granules. Histologically there are fungal druses, plasma and xanthome cells, and macrophages detected. Specify the most correct etiological name of this pathological process:

Explanation

Actinomycetes are true bacteria (related to Corynebacteria and Mycobacteria), but they form long, branching filaments that resemble the hyphae of fungi. They are gram positive but some (such as Nocardia asteroids) are also weakly acid-fast. There are two medically important organisms, Actinomyces israelii and Nocardia asteroids. Actinomyces israelii causes Actinomycosis (abscess with draining sinuses). Strictly anaerobic; it forms part of the normal flora of the oral cavity. Actinomycosis appears as a hard, nontender swelling that develops slowly and eventually drains pus through sinus tracts. In about 50% of cases, the initial lesion involves the face and neck.
69. In preparation for business trip abroad the doctor was prescribed a histoschizontocidal antimalarial drug as a personal means of disease prevention. What drug was given to the doctor?

Explanation

Chingamin (chloroquine) prevents polymerization of the hemoglobin breakdown product (heme) into hemozoin. Intracellular accumulation of heme is toxic to the parasite. It is a weak base and may buffer intracellular pH, thereby inhibiting cellular invasion by parasitic organisms. The drug is solely a blood schizonticide (erythrocytic) and will not eradicate secondary tissue schizonts. It is the drug of choice for acute attacks, prophylaxis and treatment of malaria, has been used in amebic liver disease in combination with metronidazole and in autoimmune disorders including rheumatoid arthritis, systemic lupus erythematous (interstitial disease).
70. A patient, who has been suffering from bronchial asthma for a long time, developed acute respiratory failure. What is the main mechanism of pathology development in this case?

Explanation

Obstructive respiratory disease is the abnormal respiratory condition characterized by difficulty in expiration. E.g asthma, chronic bronchitis, emphysema, cystic fibrosis. Forced expiratory volume (FEV) is the volume of air, which can be expired forcefully in a given unit of time. It is very much decreased in obstructive diseases like asthma and emphysema. Restrictive respiratory disease is the abnormal respiratory condition characterized by difficulty in inspiration. E.g poliomyelitis, myasthenia gravis, paralysis of diaphragm, spiral cord diseases, pleural effusion, fibrosis. (lung fibrosis-pneumofibrosis)
71. A 40-year-old woman has undergone thyroidectomy. Histological study of thyroid gland found the follicles to be of different size and contain foamy colloid, follicle epithelium is high and forms papillae, there is focal lymphocytic infiltration in the stroma. Diagnose the thyroid gland disease:

Explanation

 

Basedow’s disease (Grave’s disease, diffuse toxic goiter, primary hyperplasia): prismatic epithelium turns into cylindrical, epithelium proliferation with formation of papillae, colloid vacuolization (foamy colloid), formation of lymphoid follicles with germ centers are observed.

Hashimoto’s thyroiditis is devoid of colloid.

72. A patient has been hospitalized with provisional diagnosis of virus B hepatitis. Serological reaction based on complementation of antigen with antibody chemically bound to peroxidase or alkaline phosphatase has been used for disease diagnostics. What is the name of the applied serological reaction?

Explanation

Immune-enzyme analysis (enzyme-linked immunosorbent assays): used to detect the presence of either a specific antigen or a specific antibody in a patient’s blood sample. Common enzymes used are peroxidase, alkaline phosphatase or glucose oxidase. Producing an observable colour change or fluorescence. When radioactive isotopes are incorporated – Radioimmunoassay.
73. Due to blood loss the circulating blood volume of a patient decreased. How will it affect the blood pressure in this patient?

Explanation

A decrease in systolic pressure occurs due to low cardiac output and diastolic pressure, due to a reduction in vascular resistance as a result of low blood volume.

 
74. A bacteriological laboratory tests canned meat for botulinum toxin. Extract of the tested material and ABE botulinum antitoxin serum was introduced into the test group of mice; the control group of mice received the extract without antibotulinic serum. What serological reaction was used?

Explanation

Neutralization reaction are antigen-antibody reaction in which the biological effects of viruses and toxins are made ineffective by homologous antibodies.

Precipitation is a test in which antibody interacts with the soluble antigen in the presence of an electrolyte at a specified pH and temperature to produce a precipitate. 

 
75. A 50-year-old man is diagnosed with ischemic heart disease and cardiosclerosis with hypertensive syndrome. What drug should be prescribed in this case?

Explanation

 Metoprolol is a beta 1 selective blocker. Used in treating hypertension, it  decreases heart rate, contractility and cardiac output thereby reducing blood pressure.

Strophanthin and corglycon are indicated in acute heart failure.

 
76. Poisoning caused by mercury (II) chloride (corrosive sublimate) occurred in the result of safety rules violation. In 2 days the patient’s diurnal diuresis became 620 ml. The patient developed headache, vomiting, convulsions, dyspnea; moist crackles are observed in the lungs. Name this pathology:

Explanation

Acute tubular necrosis/Necrotic nephrosis/Necronephrosis involves the death of tubular epithelial cells that form the renal tubules of the kidneys. Most common cause of acute renal failure. It can be ischemic or nephrotoxic.

·        Ischemic acute tubular necrosis occurs due to hypoperfusion of the kidneys.

·        Nephrotoxic acute tubular necrosis occurs as a result of direct damage to tubular cells by ingestion, injection or inhalation of a number of toxic agents. Toxic agents causing nephrotoxic acute tubular necrosis includes mercuric chloride, ethylene glycol, carbon tetrachloride etc.

Macroscopically, the kidneys are enlarged and swollen (edematous). On cut section, the cortex is pale, while the medulla is slightly darker than normal. The capsule can be easily removed.

77. Autopsy of a 9-year-old child shows numerous irregular defects of varying depth with uneven margins and gray-white films tightly attached to the underlying tissue on the rectal mucosa of the body. What disease can be suspected?

Explanation

Dysentery refers to bloody diarrhea with mucus. It refers to diarrhea with abdominal cramping and tenesmus in which loose stools contain blood, pus and mucus. Bacillary dysentery is caused by shigella dysenteriae, Shigella flexneri, Shigella boydii and Shigella sonnei. Morphology: it has 4 stages namely: catarrhal colitis; fibrinous colitis; ulcer formation (ulcerative colitis); healing of the wound. Complications of dysentery includes perforation, intraintestinal hemorrhage and eventually death may result from intestinal or extraintestinal complications.

Fibrinous colitis stage produces a dirty grayish pseudomembrane (film). Catarrhal colitis stage produces edema (inflamed) and hyperemic mucosa.

78. A man presents with glomerular filtration rate of 180 ml/min., while norm is 125±25 ml/min. The likely cause of it is the decreased:

Explanation

 Glomerular filtration rate (GFR) is the volume of fluid filtered from the renal (kidney) glomerular capillaries into the Bowman's capsule per unit time. Factors affecting GFR include;

  1. Renal blood flow rate

  2. Glomerular capillary hydrostatic pressure

  3. Oncotic pressure

  4. Glomerular capillary permeability etc

Reduced oncotic pressure leads to edema ( accumulation of fluid).

 
79. A young man came to a hospital with complaints of disturbed urination. Examination of his external genitalia revealed the urethra to be split on the top, with urine flowing out of this opening. What type of external genitalia maldevelopment is observed in this case?

Explanation

Defects in male genitalia:

Hypospadia: fusion of the urethral folds is incomplete and abnormal openings of the urethra occur along the inferior (undersite) surface of the penis, usually near the glans, along the shaft or near the base of the penis.

Epispadia is a rare abnormality (1/30000 births) in which the urethral meatus is found on the dorsum (superior) surface of the penis.

80. A 10-year-old child had cut his leg with a glass shard, when playing, and was delivered to the outpatient department to receive antitetanus serum. To prevent development of anaphylactic shock the serum was introduced by Bezredka method. This method of organism hyposensitization is based on the following mechanism:

Explanation

Desensitization as a method of treating allergic diseases is used, in particular, to prevent serum anaphylaxis from repeated injections of serum preparations (e.g. tetanus toxoid), in accordance with the method proposed in 1907 by the Russian scientist A. M. bezredka. The method consists in injecting small concentrations of the preparation (toxoid antigen) that produce the sensitization, as a result, a state of anti-anaphylaxis arises (i.e. desensitization). Owing to this, the next injection of the reacting dose of the allergen does not produce anaphylaxis. The small concentrations injected are below threshold concentrations inducing complete and sustained deprivation of releasability (ability to release inflammatory mediators from activated mast cells having IgE attached to them already) without triggering degranulation. Basophil desensitization is regulated by their surface IgE levels.

Desensitization therapy in atopic individuals involves repeated injections of increasingly greater amounts of allergens, resulting in production of IgG antibodies that attach to allergens and prevent them from binding to mast cells.

81. A 38-year-old man, who has been suffering from systemic lupus erythematosus for 3 years, developed diffuse renal lesions accompanied by massive edemas, marked proteinuria, hyperlipidemia, and dysproteinemia. What is the most likely mechanism of proteinuria develoment in this case?

Explanation

Systemic lupus erythematous (SLE, Libman-sacks disease) is the classic prototype of the multisystem disease of autoimmune origin, characterized by a bewildering array of autoantibodies, particularly antinuclear antibodies. It is characterized principally by injury to the skin, joints, kidney, and serosal membranes. Antinuclear antibody is directed against several nuclear antigens and can be grouped into 4 categories:

·        Antibodies to DNA

·        Antibodies to histones

·        Antibodies to nonhistone proteins bound to RNA

·        Antibodies to nuclear antigens.

SLE is a type III hypersensitivity reaction (autoimmune - immune complex) with formation of immune complexes. It can cause diffuse proliferative glomerulonephritis seen under the light microscope as “wire loop” of capillaries and granular under the immunofluorescence microscopy.

82. During experiment a part of the brain was extracted, which resulted in asynergy and dysmetria development in the test animal. What part of the brain was extracted in the animal?

Explanation

During cerebellar lesions, there  are disturbances in posture, equilibrium and movements. Disturbances in movements: Speech disorders, ataxia, asynergia, asthenia (weak muscle contractions with characteristically rapid onset of muscle fatigue), Dysmetria, Intention tremor, astasia (loss of the capacity for sustained tetanic contractions), nystagmus, rebound phenomenon, dysarthria, adiadochokinesis, atonia (lack or impairment of muscle tone).

NB: Scanned or staccato speech is as a result of cerebellar disorders; while monotonus speech is as a result of parkinson\'s disease.

83. A woman with polyarticular rheumatoid arthritis was prescribed a non-steroidal antiinflammatory drug - diclofenac sodium. After the patient has been taking it for some time, her concomitant disease exacerbated, which forced the doctor to cancel the prescription of this drug. What concomitant disease could necessitate cancellation of this drug prescription?

Explanation

 

Diclofenac sodium is a non-steroidal anti-inflammatory drug. A prolonged use may lead to development of ulcers in the stomach or intestine.

84. Histologic preparation stained with orcein demonstrates from 40 to 60 fenestrated elastic membranes within the middle coat of the vessel. Name this vessel:

Explanation

All blood vessels except capillaries have 3 basic tunics or coats: -Tunica Intima (Inner coat) -Tunica Media (middle coat) -Tunica Adventitia (Outer coat) The following are visible in the Tunica Media of elastic arteries stained with orcein: - Elastic tissue in the form of 40 - 70 fenestrated elastic membranes. - Smooth muscles and collagen fibers in matrix - A layer of elastic fibers external elastic lamina forms the boundary between Tunica media and Tunica adventitia Presence of elastic fibers in the wall of elastic arteries allows it to expand during contraction (systole) and recoil during relaxation (diastole).
85. A woman with the III (В), Rh (-) blood group gave birth to a child with the II (А) blood group. The child is diagnosed with hemolytic disease of newborn caused by rhesus incompatibility. What blood group and Rh are likely in the father?

Explanation

Rhesus incompatibility is observed when the mother possesses an Rh -ve blood group and the developing child has an Rh +ve blood group (most likely gotten from the father).
86. A 40-year-old woman with Cushing’s disease presents with steroid diabetes. On biochemical examination she has hyperglycemia and hypochloremia. What process activates in the first place in such patients?

Explanation

Cushing's syndrome is characterized by increased glucocorticoids. Glucocorticoids (prednisolone) stimulate gluconeogenesis. As a result, blood sugar rises, muscle protein is catabolized and insulin secretion is stimulated. Corticosteroids cause increased gluconeogenesis, increased lipolysis, CNS effects - at times including euphoria, maintenance of cardiovascular function by potentiation of norepinephrine. Gluconeogenesis uses the reversible reactions from glycolysis and 4 distinct reactions that circumvent the ones from glycolysis that are irreversible. These reactions are catalyzed by pyruvate carboxylase, phosphoenolpyruvate carboxykinase, fructose-1, 6-bisphosphatase and glucose-6-phosphatase. Glucagon, epinephrine and glucocorticoids stimulate all these enzymes (excluding pyruvate carboxylase).
87. A 40-year-old prisoner died of tuberculosis in the corrective labor camp. Autopsy of the body revealed deformation and diminishing of both lung apices; in the both upper lobes there are multiple cavities with dense walls 2-3 mm thick; in the lower lung lobes there are disseminated foci of caseous necrosis varying from 5 mm to 2 cm in diameter. Diagnose the type of tuberculosis:

Explanation

In secondary fibrous-cavernous/(cavitary) tuberculosis,there are presence of disseminated foci of caseous necrosis in the lower lung lobe. Walls of the cavity has three membranes:

Inner: contains necrotic tissue

Middle: contains special granular tissue

External: contains fibrotic tissues.

 
88. A 7-year-old boy died of acute posthemorrhagic anemia caused by profuse hemorrhage in the gastrointestinal tract. Postmortem study revealed the following: macroscopically there were acutely enlarged various groups of the lymph nodes, thymomegaly, hepatosplenomegaly, and bright red bone marrow; microscopically there was hypercellular bone marrow with monomorphic infiltrations composed of blasts and diffuse-focal tumor infiltrations in the liver, spleen, lymph nodes, brain substance and tunics. Make the diagnosis:

Explanation

In acute lymphoblastic leukemia, infiltrations are made up of lymphoblastic cells.

 
89. Autopsy of a man, who served on a nuclear submarine, revealed the following pathologies: bone marrow atrophy (panmyelophthisis), anemia, leukopenia, thrombocytopenia, lymphocytes disintegration in the lymph nodes, spleen, gastrointestinal lymphatic system, and hemorrhages into the adrenal glands. What disease had developed in this case?

Explanation

 

Acute radiation sickness occurs due to exposure to radiation. Symptoms include, aplastic anemia, leukopenia, thrombocytopenia, tumors etc.

Decompression sickness  refers to injuries caused by a rapid decrease in the pressure that surrounds you, of either air or water. It occurs most commonly in scuba or deep-sea divers, although it also can occur during high-altitude or unpressurized air travel.

90. After sensitization a test animal received subcutaneously a dose of antigen. At the site of injection a fibrinous inflammation developed with alteration of vessel walls, basal substance, and fibrous structures of connective tissue. The inflammation took form of mucoid and fibrinoid degeneration, fibrinoid necrosis. What immune response occurred in the test animal?

Explanation

Type I (Immediate, Anaphylaxis, Reagin): IgE (immunoglobulin E)-dependent activation of mast cells/basophils, usually accompanied by eosinophilia e.g. urticaria (hives), hay fever, asthma (wheezing), rhinitis and conjunctivitis (stuffy nose and itchy eyes; usually seasonal)
91. Preoperative examination revealed prothrombin deficiency in the blood of the patient. What drug should be preliminarily prescribed to mitigate blood loss in the patient during the surgery?

Explanation

Vicasol or Menadione is a drug used as a precursor for vitamin K. It is indicated in bleeding/hemorrhage.

Contrycal is indicated in acute pancreatitis.

 
92. A patient complaining of pain in the left shoulder-blade region has been diagnosed with miocardial infarction. What kind of pain does the patient have?

Explanation

* Radiating pain: perceived at a site adjacent to or away from the site of origin but in the same dermatome i.e. supplied by afferent nerve fibers of one dorsal root.

Viscera pain: organs, poorly localized, diffuse

Protopathic pain: poorly localized pain

Epicritic pain: well localized pain

Phantom pain: pain from non-existent body structures (amputated limbs)

Phantom – if a leg has been amputated, the cut end heals with scar formation. The cut ends of nerve fibers are merged within the scar. If the cut end of sensory fibers are stimulated during movement of thigh, the patient feels as if the sensation is originating from non-existent leg. Sometimes the patient feels pain in non-existent limb.

93. During regular check-up a child is determined to have interrupted mineralization of the bones. What vitamin deficiency can be the cause?

Explanation

 

Vitamin D is a fat soluble vitamin. Cholecalciferol (Vit. D3) is found in animals. It is regarded as a sunshine vitamin. Calcitriol is the biologically active form of Vit. D. Calcitriol acts at 3 different levels (intestine, kidney and bone) to maintain plasma calcium.

It increases the intestinal absorption of calcium and phosphate.

In the osteoblasts of bone, it stimulates calcium uptake for deposition as calcium phosphate. Thus, calcitriol is essential for bone formation.

Calcitriol is also involved in minimizing the excretion of calcium and phosphate through the kidney, by decreasing their excretion and enhancing reabsorption.

Deficiency of Vitamin D causes rickets in children and osteomalacia in adults. Rickets in children is characterized by bone deformities due to incomplete bone mineralization, resulting in soft and pliable bones and delay in teeth formation. The weight bearing bones are bent to form bow legs.

94. On examination the patient was determined to have strong, balanced, inert type of higher nervous activity according to Pavlov’s classification. What temperament according to Hippocrates is it?

Explanation

Hippocrates classification due to temperament and social behavior: choleric, phlegmatic; sanguine; melancholic.

·        Choleric: egocentric and extroverts, excitable, impulsive and restless (dry and hot).

·        Phlegmatic: reasonable, calm, patient, caring and tolerant (moist and cold)

·        Sanguine: lively, sociable, carefree, talkative and pleasure seeking (hot and moist)

·        Melancholic: serious, introvert, cautious or even suspicious (cold and dry)

95. A specimen shows an organ covered with connective tissue capsule with trabeculae radiating inward the organ. The organ’s cortex contains lymph nodules; there are medullary cords made of lymphoid cells. What organ is under study?

Explanation

Lymph nodes are small encapsulated organs located along the pathway of lymphatic vessels. The supporting elements of the lymph node are:

 *Capsule: composed of dense connective tissue that surrounds the node.

*Trabeculae: composed of dense connective tissue, which extend from the capsule into the substance of the node, forming a gross framework.

*Reticular tissue: composed of reticular cells and reticular fibers that forms a fine supporting meshwork throughout the remainder of the organ.

The parenchyma of the lymph node is divided into a cortex and medulla. The cortex consists of lymphocytes which are organized into nodules. The medulla of the lymph node consists of the medullary cords and medullary sinuses.

Tonsils: form a ring of lymphatic tissue at the entrance of the oropharynx; consisting of aggregations of lymphatic nodules.

Thymus is a lymphoepithelial organ located in the superior mediastinum. It possess a thin connective tissue capsule from which trabeculae extend into the parenchyma of the organ. The trabeculae establish domains in the thymus called thymic lobules. Thymic or hassall’s corpuscles are a distinguishing feature of the thymic medulla.

Spleen is the largest lymphatic organ. Most of the spleen consists of splenic pulp. Splenic pulp is divided into white pulp and red pulp. The spleen is enclosed by a dense connective tissue capsule from which trabeculae extend into the parenchyma of the organ.

Red bone marrow lies entirely within the spaces of bone in the medullary cavity of young long bones and the spaces of spongy bone.
96. Brain autopsy revealed an edema, hyperemia, and small hemorrhages in the medulla oblongata. Microscopically chromatolysis, hydropia and nerve cell necrosis are observed; within the cytoplasm of hippocampal nerve cells there are eosinophilic structures (Negri bodies) detected. What diagnosis corresponds with the described morphological signs?

Explanation

 Babes-Negri bodies are found in rabies; Paschen bodies are found in Variola (smallpox).

 
97. A married couple came for a genetic counseling. The husband suffers from insulin-independent diabetes mellitus, while the wife is healthy. What is the probability of their child developing insulin-independent diabetes mellitus?

Explanation

A child from a diabetic parent has a higher probability of being diabetic in future.

 
98. A 46-year-old patient suffering from ulcer disease of the stomach is diagnosed with rheumatoid arthritis. What antiinflammatory drug would be the most advisable in this case?

Explanation

 

The nonsteroidal anti-inflammatory drugs (NSAIDs) are a group of chemically dissimilar agents that differ in their antipyretic, analgesic and anti-inflammatory activities. They act primarily by inhibiting the cyclooxygenase (COX 1 and 2) enzymes that catalyze the first step in prostanoid biosynthesis. This leads to decrease prostaglandin synthesis with both beneficial and unwanted effects. Aspirin is one of the most important NSAID. Its most common side effect is its gastrointestinal (GI) effect. Normally, prostacyclin (PGI2) inhibits gastric acid secretion, whereas PGE2 and PGF stimulate synthesis of protective mucus in both the stomach and small intestine.

Celecoxib is a selective cyclooxygenase 2 (COX-2) inhibitor. Approved for treatment of rheumatoid arthritis, osteoarthritis, acute to moderate pain, also approved for patients with peptic ulcers or gastroduodenitis. This is due to the fact that only COX 1 is responsible for the gastrointestinal protective functions.

99. Electron micrograph of the kidney shows fenestrated endothelium lying on the basement membrane; the external surface of the membrane has adjacent dendritic epithelial cells. What do these structures form in the kidney?

Explanation

The filtration barrier of the glomerulus consists of fenestrated epithelium lying on basement membrane also known as filtration pores.

Proximal part of nephron has cuboidal epithelium with brush borers.

 

Distal part has no brush border.

100. A worker of an agricultural enterprise had been suffering from an acute disease with aggravating intoxication signs, which resulted in his death. On autopsy: the spleen is enlarged, flaccid, dark cherry-red on section, yields excessive pulp scrape. Soft meninges of the fornix and base of the brain are edematous and saturated with blood (”cardinal’s cap”). Microscopically: seroushemorrhagic inflammation of meninges and cerebral tissues. Make the diagnosis:

Explanation

There are two medically important Bacillus species: Bacillus anthracis and Bacillus cereus. Bacillus anthracis causes anthrax. Human disease occurs in 3 main forms: cutaneous, pulmonary (inhalation) and gastrointestinal. Humans are most often infected cutaneously at the time of trauma to the skin, which allows the spores on animal products such as hides, bristles and wool to enter. Spores can also be inhaled or when contaminated meat is ingested. After being inhaled, the organism moves rapidly to the mediastinal lymph nodes and causes hemorrhagic mediastinitis. Pathogenesis is based on the production of two exotoxins  (Anthrax toxin) – edema factor and lethal factor. Hemorrhagic mediastinitis, septic shock hemorrhagic meningitis and death are severe life-threatening complications. In fatal cases, the organism may affect the spleen, liver, intestines, kidneys, adrenal glands and meninges.
101. A woman suffers from tonsillitis complicated with retropharyngeal abscess that is localized in the spatium retroviscerale. In this case the suppurative process can spread to the:

Explanation

 

The mediastinum is the space between the two lungs in the thoracic cavity. It is divided into Superior and Inferior ( anterior, middle and posterior). A retro pharyngeal abscess is likely to spread to the posterior mediastinum.

102. A patient with chronic bronchitis was prescribed a drug with mucolytic action. Name this drug:

Explanation

A mucolytic drug depolymerizes ( breakdown) mucus. Ambroxol and acetylcysteine belong to this group. Anaprillin is a non-selective beta blocker and Atropine is a muscarinic blocker.

 
103. A patient suffers from acute cardiopulmonary failure with pulmonary edema. What diuretic should be prescribed in the given case?

Explanation

Bumetanide, furosemide, torsemide, ethacrynic acid are diuretics that have their major action on the ascending limb of the loop of henle (loop diuretics). These drugs are useful in emergency situation which calls for a rapid, intense dieresis (forced dieresis). Loop or high-ceiling diuretics inhibit the cotransport of Na+/K+/2Cl- on the luminal membrane in the ascending limb of the loop of henle.
104. Therapeutics unit of a hospital received a man suffering from ulcer disease of the stomach with hyperacidity. Which of the listed group of drugs must be used as a part of the complex therapy of this patient?

Explanation

Histamine H2 receptors are found on the walls of the stomach, when stimulated, they lead to the release of hydrochloric acid thereby increasing acidity. Examples of H2 blockers include famotidine, ranitidine etc. H1 Blockers (eg. Loratadine) are used in allergic diseases such as rhinitis.
105. A 45-year-old woman gave birth to a boy with cleft maxilla (cleft lip and palate). On additional examination there are significant disturbances of the boy’s nervous, cardiovascular, and visual systems. Karyotype investigation allowed diagnosing the patient with trisomy 13. What syndrome is present in the boy?

Explanation

Trisomy 13 (Patau’s syndrome): the main abnormalities of this syndrome are mental retardation, holoprosencephaly, congenital heart defects, microcephaly, polydactylyl, microphthalmus, anophthalmos, cleft lip and palate (cheiloschisis and uranoschisis respectively).
106. Pathologic material (mucosal excretion from the nasal passages) obtained from a patient provisionally diagnosed with influenza was delivered to the virological laboratory. What quick test allows detecting specific viral antigen in the investigated material?

Explanation

Immunofluorescence (fluorescent antibody): fluorescent dyes e.g. fluorescein and rhodamine, can be covalently attached to antibody molecules and made visible by UV light in the fluorescence microscope. The immunofluorescence reaction is DIRECT when known labeled antibody interacts directly with unknown antigen and INDIRECT when a two-stage process is used e.g. known antigen is attached to a slide, the patient’s serum (unlabelled) is added and the preparation is washed; if the patient’s serum contains antibody against the antigen, it will remain fixed to it on the slide and can be detected on addition of a fluorescent dye-labelled antibody to human IgG and examination by UV microscopy. The indirect test is often more sensitive than direct immunofluorescence, because more labeled antibody adheres per antigenic site. Immunofluorescence staining of cells obtained from the patient or of cells infected in culture can provide a rapid and specific diagnosis.
107. A 5-year-old child is diagnosed with Bruton syndrome (X-linked agammaglobulinemia) that manifests itself in severe clinical course of bacterial infections and absence of B lymphocytes and plasma cells. What changes of immunoglobulin content can be observed in blood serum of the child with immunodeficiency?

Explanation

Bruton’s agammaglobulinemia is an X-linked immunodeficiency disease that occurs due to the mutation in the gene that codes Bruton tyrosine kinase enzyme (BTK). This enzyme helps B lymphocytes in maturing. Since B cells are responsible for the production of immunoglobulins, a reduction in antibody levels are observed especially IgA and IgM levels.

 
108. A 37-year-old man, who was working in a caisson, after being lifted to the surface suddenly developed signs of acute cerebral circulation disturbance and loss of consciousness. Several days later he died. On autopsy in the left cerebral hemisphere there was detected a gray soft irregular focus 5х6х3,5 cm in size. What process had occurred in the brain?

Explanation

Stroke is sudden loss of blood circulation to an area of the brain resulting in a corresponding loss of neurologic function. Types:

·        Ischemic (70-80% of cases):

- Atherosclerotic (thrombotic): most common type

- Embolic

·        Intracerebral hemorrhage

·        Subarachnoid hemorrhage

·        Lacunar stroke

Atherosclerotic (thrombotic) stroke: Ischemic type of stroke is commonly caused by platelet thrombosis that develops over a disrupted atherosclerotic plaque. Characteristics: it usually develops at the periphery of the cerebral cortex, swelling of brain, loss of dermacation between gray and white matter, breakdown of myelin, pale infarct; Cystic area develops after 10days to 3weeks due to liquefactive necrosis.

109. A short-term physical load resulted in reflex increase of heart rate and systemic arterial pressure in a person. What receptor activation was the most contributory to inducing the pressor reflex?

Explanation

Proprioreceptors are the receptors which give response to change in the position of the body. These receptors are situated in joints, tendons and muscles. Proprioreceptors are stimulated during the muscular exercise and send impulses to the brain, particularly cerebral cortex, through somatic afferent nerves. Cerebral cortex in turn causes increase in heart rate, force of contraction of heart, arterial pressure, hyperventilation by sending impulses to medullary vasomotor and respiratory centers.
110. During experiment a skeletal muscle is being stimulated with a series of electrical impulses. What type of muscular contraction will develop, if each following impulse occurs within the relaxation period after the previous single contraction of the muscle?

Explanation

 

Tetanus is defined as the sustained contraction of muscle due to repeated stimuli with high frequency. When the multiple stimuli are applied at a higher frequency in such a way that the successive stimuli fall during contraction period (period of shortening) of previous twitch, the muscle remains in state of tetanus. While increasing the frequency, fusion of contractions increases every time and finally complete tetanus occurs. Holotetanus – complete tetanus.

When the frequency of stimuli is not sufficient to cause tetanus, the fusion of contractions is not complete and then we have – incomplete/partial/waved tetanus. This occurs (i.e. the next stimuli occurs) during the period  of relaxation.

111. Depression and emotional disturbances result from the lack of noradrenaline, serotonin, and other biogenic amines in the brain. Their content in the synapses can be increased through administration of antidepressants that inhibit the following enzyme:

Explanation

  Norepinephrine (noradrenaline), epinephrine (adrenaline) and dopamine are catecholamines produced in chromaffin cells of adrenal medulla from tyrosine. The catecholamines are inactivated by oxidative deamination catalyzed by monoamine oxidase (MAO) and by O-methylation carried out by Catechol-O-methyltransferase. The metabolic products of these reactions are excreted in the urine as vanillymandelic acid from epinephrine and norepinephrine; and homovanillic acid from dopamine.
112. A 2-year-old child presents with acute psychomotor retardation, vision and hearing impairment, sharp enlargement of the liver and spleen. The child is diagnosed with hereditary Niemann-Pick disease. What genetic defect is the cause of this disease?

Explanation

Niemann-pick disease is a lipid storage disease characterised by accumulation of sphingomyelin in lysosomes. Occurs due to dysfunction of sphingophospholipid metabolism. In Type A and B there is a deficiency of the enzyme sphingomyelinase an enzyme necessary for the formation sphingomyelins.

Glucose-6-phosphatase is deficient in von gierke’s disease.

113. Clinical presentations of a woman allowed provisionally diagnosing her with X polysomy. Cytogenetic method is applied to clarify the diagnosis. The diagnosis will be confirmed if the patient’s karyotype is:

Explanation

Barr body is an inactive X-chromosome. A normal female has one barr body XX, a normal male has no barr body XY.

Trisomy X – XXX (only one X is active in a female; therefore, 2 barr bodies)

Klinefelter – XXY (one barr body)

Turner’s – XO (no barr body)

46 XX is a normal female sex chromosome. All other options are karyotypes for male patients.

114. During dehelmintization there was a 3,5-meter-long tapeworm produced from the patient’s intestine.There are 4 suckers and hooks on the tapeworm’s scolex. Mature segments of the tapeworm are immobile and have up to 12 uterine branches. What disease is it?

Explanation

 

Taeniasis is a disease caused by taenia solium ( 7-112 uterine branches) or taenia saginata (17-35).

Diphyllobothriasis is caused by diphyllobothrium latum. In opisthorchiasis, we see the presence of small/operculate eggs.

115. A patient presents with steatorrhea. This disorder can be linked to disturbed supply of the intestine with the following substances:

Explanation

Steatorrhea is the formation of bulky, foulsmelling, frothy and clay-coloured stools with large quantity of undigested fat because of impaired digestion and absorption of fat. Any condition that causes indigestion or malabsorption of fat leads to steatorrhea. Causes of steatorrhea are: lack of pancreatic lipase, liver disease affecting the secretion of bile (lack of bile acid or obstruction of bile duct), celiac disease and cystic fibrosis.
116. Cytochrome oxidase is a hemeprotein that is an end component of the mitochondrial respiratory chain. What reaction is catalyzed with this enzyme?

Explanation

 

Cytochrome c oxidase catalyses the last reaction in the electron transport chain. It receives an electron from each of four cytochrome c molecules, and transfers them to one dioxygen molecule, converting the molecular oxygen to two molecules of water. Cytochrome P450 oxidises xenobiotics, steroids and proteins.

117. A 16-year-old girl fainted when she tried to quickly change her position from horisontal to vertical. What caused the loss of consciousness in the girl?

Explanation

From horizontal (lying position) to vertical (standing position) will make all blood flow towards the lower extremities, thereby reducing venous return (blood going upwards towards the heart) in the first place. This will definitely ↓ cardiac output (C.O) → ↓blood supply to brain (this can lead to fainting). Then there is reflex vasoconstriction to increase venous return to heart, in the absence of any pathology.
118. An unconscious patient was delivered by ambulance to the hospital. On objective examination the patient was found to have no reflexes, periodical convulsions, irregular breathing. After laboratory examination the patient was diagnosed with hepatic coma. Disorders of the central nervous system develop due to the accumulation of the following metabolite:

Explanation

Substances absorbed into the bloodstream from the intestine pass through the liver, where toxins are normally removed. Many of these toxins (such as ammonia) are normal breakdown products of the digestion of protein. In hepatic encephalopathy (hepatic coma), toxins are not removed because liver function is impaired. Ammonia is produced by amino acid metabolism and intestinal urease-positive bacteria. In physiological conditions, it is mostly present as ammonium (NH4+) in serum. The urea or ornithine cycle, which is fully expressed in the liver exclusively, serves to converts NH4+ to urea prior to renal excretion and to maintain low serum concentrations. In hepatic coma, when the liver cannot remove toxins and urea cycle is not functional, all this occurs:

NH3 + α-ketoglutarate → Glutamate

α-ketoglutarate is used up which leads to:

·        ↑glutamate → ↑GABA (inhibitory neurotransmitter)

·    Inhibition of citric acid cycle/tricarboxylic acid cycle; this causes impairment of ATP formation.

·        Inhibition of metabolism of amino acids (impairment of transamination reactions).

NH3 + Glutamate → Glutamine

Glutamine is an amide of glutamic acid which provides a non-toxic storage and transport form of ammonia (NH3). Ammonia increase synthesis of glutamine in brain. Accumulation of glutamine in brain results in elevation of osmotic pressure in nervous cells leading to brain edema.

                                    NH3 + H+ → NH4+

In blood ammonia (NH3) is represented as ammonium ion (NH4+). Accumulation of ammonium ion impairs transport of ions (Na+, K+) through cell membranes and failure of transmission of nerve impulse.

Urea cycle takes place exclusively in the liver, so in hepatic coma, urea level is low. Glutamine toxicity in brain is dependent on increased ammonia concentration.

Bilirubin toxicity will most likely be related to increase hemolysis, which is not the case in this question. Histamine is a biogenic amine produced from the amino acid histidine.

119. A patient complains of acute pain attacks in the right lumbar region. During examination the nephrolithic obturation of the right ureter in the region between its abdominal and pelvic segments has been detected. What anatomical boundary exists between those two segments?

Explanation

 

Linea terminalis = pectineal line (pubis) + Arcuate line + sacral promontory + superior margin of pubic symphysis. Boundary between the abdominal and pelvic cavity.

Linea semilunaris found on the lateral margin of rectus abdominis.

Linea arcuata: the region on the posterior layer, where aponeuroses end and continue into the anterior layer. Part of linea terminalis anteriorly.

120. After mushroom poisoning the patient developed signs of acute hepatic failure leading to his death. On autopsy the liver is diminished, flaccid; the capsule is wrinkled; the tissue is ochre-yellow on section. Microscopically: fatty degeneration of hepatocytes, necrotic central segments of the hepatic lobes. These changes are characteristic of:

Explanation

A degeneration of fats in the liver accompanied by necrotic central segment in hepatic lobes indicates a massive progressive necrosis. In fatty hepatosis, we see fat deposition in the liver.
121. Blood test of an athlete shows the following: erythrocytes - 5, 5 · 1012/l, hemoglobin - 180 g/l, leukocytes - 7 · 109/l, neutrophils - 64%, basophils - 0,5%, eosinophils - 0,5%, monocytes - 8%, lymphocytes - 27%. These values primarily indicate the stimulation of:

Explanation

        Normal range

               Result

RBC (erythrocytes): Men

      4.2 – 5.2 X 1012/L

          5.5 X 1012/L     (↑)

                               Women

      3.7 – 4.7 X 1012/L

 

Hemoglobin:  Men

      135 – 165g/L

           180g/L        (↑)

                        Women

      120 – 140g/L

           

Leukocytes (WBC):

      4.0 – 9.0 X 109/L

      7 X 109/L    (normal)

    Basophils

         0 - 1%

           0.5%    (normal)

    Eosinophils

         0.5 – 5%

           0.5%    (normal)

    Neutrophils

         47 – 72%

           64%     (normal)

    Lymphocytes

         19 – 37%

           27%     (normal)

    Monocytes

           3 – 11%

            8%      (normal)

 

This results indicates erythropoiesis i.e. synthesis of RBCs.

122. Experimental stimulation of the sympathetic nerve branches that innervate the heart caused an increase in the force of heart contractions because the membrane of typical cardiomyocytes permitted an increase in:

Explanation

  When a muscle is excited (stimulated) by the impulses passing through neuromuscular junction, action potential is generated which spreads over sarcolemma (plasma membrane of muscles). When the action potential reaches the cisternae of ‘L’ tubules, Ca2+ stored in the cisternae are released into the sarcoplasm (cytoplasm of muscles). The Ca2+ moves towards the actin filaments to produce the contraction. Therefore, it is Ca2+ entry into the sarcoplasm.
123. As a result of past encephalitis, a man has developed an increase in cerebrospinal fluid pressure in the right lateral ventricle. What can be the cause of this condition?

Explanation

    Encephalitis is inflammation of the brain. 4 ventricles containing cerebrospinal fluid (CSF) within the brain. CSF is formed by the choroid plexus, within the cerebral ventricles. CSF moves from the lateral ventricles (right and left) → interventricular foramen (of Monro) → 3rd ventricle → cerebral aqueduct (of sylvius) → 4th ventricle. An ↑ in CSF pressure in right lateral ventricle is due to obstruction or closure of the right interventrcular foramen preventing outflow of CSF.
124. A patient with pneumonia has body temperature of 39,2oC. What cells are the main producers of endogenous pyrogen that had caused such temperature rise?

Explanation

Monocytes have mainly 3 functions; phagocytosis, antigen presentation and cytokine production. Typical cytokines produced by monocytes are TNF & IL-1 ( pyrogenic cytokines) , and IL-12.
125. A patient is diagnosed with compression fracture of the lumbar vertebra. The patient presents with acutely increased lumbar lordosis. What ligament was damaged in this patient resulting in such deformation of vertebral column curvature?

Explanation

Lordosis is an exaggerated anterior curvature of the spine, most often lumbar. Lordosis is present in the cervical and lumbar regions (cervical and lumbar lordoses). The intervertebral synchondroses and symphyses are reinforced by the longitudinal ligaments which run along the entire spine.

·        Anterior longitudinal ligament is a band which extends from the atlas to the pelvic surface of the hip bone along the anterior surfaces of the vertebral bodies.

·        Posterior longitudinal ligament runs along the posterior surface of the vertebral bodies (in the vertebral canal).

Therefore, a damage to the anterior longitudinal ligament can induce lordosis.

126. Gram-positive spore-forming bacilli were extracted in anoxic environment from the patient’s wound contaminated with soil. Cultivation on a blood-glucose agar resulted in growth of the colonies surrounded with hemolysis zone. What agent was extracted from the wound?

Explanation

Clostridium perfringens is a gram positive, spore forming  bacillus cultivated on blood glucose agar. It causes food poisoning. Staphylococcus aureus is a gram positive spore forming bacteria (cocci) arranged in clusters, ferments mannitol and lecithinase positive.

 
127. The patient’s ECG shows that in the second standard lead from the extremities the P waves are positive, their amplitude is 0,1 mV (norm is 0,05-0,25 mV), duration - 0,1 seconds (norm is 0,07-0,10 seconds). It can be concluded that the following process occurs normally in the cardiac atria:

Explanation

 

P-wave indicates Atrial depolarization, QRS complex- ventricular depolarization, T-wave ventricular repolarization.

128. Autopsy of a patient, who died of bilateral bronchopneumonia, shows in the left lung lower lobe a cavity 5 cm in diameter, filled with liquid yellowish-white substance. What complication of the patient’s pneumonia had developed?

Explanation

Abscess is a cavity containing purulent exudate. Sequestrum- fragments of dead tissue, which can be autolized. Gangrene is an area of ischemic necrosis.

 
129. A patient, who has been subsisting exclusively on polished rice, has developed polyneuritis due to thiamine deficiency. What substance is an indicator of such avitaminosis, when it is excreted with urine?

Explanation

There is a severe thiamine -deficiency syndrome found in areas where polished rice is the major component of the diet. The oxidative decarboxylation of pyruvate and alpha-ketoglutarate, which plays a key role in energy metabolism of most cells is particularly dependent on thiamine (Vit. B1). In thiamine deficiency, the activity of the enzymes necessary to decarboxylate pyruvate is significantly decreased, resulting in decreased production of ATP and, thus, increased pyruvic acid levels which eventually gets into the urine.   Thiamine pyrophosphate (TPP) is the biologically active form of Vitamin B1.
130. It is known that pentose-phosphate pathway actively functions in the erythrocytes. What is the main function of this metabolic pathway in the erythrocytes?

Explanation

 

Erythrocytes possess no nucleus, the main metabolic pathway in these cells is the pentose phosphate pathway in which NADPH is produced in the process. In lipid peroxidation, reactive oxygen species are activated (bad oxygen that kills cells). NADPH serves as the ultimate donor of reductive power for the large majority of ROS-detoxifying enzymes thereby counteracting lipid peroxidation.

131. Pupil dilation occurs when a person steps from a light room into a dark one. What reflex causes such a reaction?

Explanation

Pupillary response is a physiological response which entails constriction or dilation of the pupil in relation to bright or low light respectively, that is the pupil dilates in a dark room and constrict in bright light. In low light conditions, as in a dark room, a dilated pupil lets more light into the eye. Constriction of the pupil is controlled by the parasympathetic nervous system while pupillary dilation is controlled by the sympathetic nervous system. Therefore, it is unconditioned (physiological, not learnt, present in everyone) and the pupil dilation is a sympathetic process.
132. A patient complaining of heartburn has undergone biopsy of the gastric mucosa. In the sample there are numerous cells with oxyphilic cytoplasm in the glandular epithelium. Name these cells:

Explanation

Parietal (oxyntic) cells are found in the neck of the fundic glands, among the mucus neck cells and in the deeper part of the gland. They secrete HCl and intrinsic factor. When examined with the transmission electron microscope they are seen to have an extensive intracellular canalicular system that communicates with the lumen of the gland. Also an elaborate tubulo-vesicular membrane system is present in the cytoplasm adjacent to the canaliculi. Numerous mitochondria with complex cristae and many matrix granules supply the high levels of energy necessary for acid secretion.
133. Inherited diseases, such as mucopolysaccharidoses, manifest in metabolic disorders of connective tissue, bone and joint pathologies. The sign of this disease is the excessive urinary excretion of the following substance:

Explanation

The Mucopolysaccharidoses are hereditary disorders that are clinically progressive. They are characterized by accumulation of glycosaminoglycans in various tissues, causing varied symptoms such as skeletal and extracellular matrix deformities and mental retardation. Mucopolysaccharidoses are caused by a deficiency of any one of the lysosomal hydrolases normally involved in the degradation of heparin sulfate and/or dermatan sulfate. This results in the presence of oligosaccharides in the urine because of incomplete lysosomal degradation of glycosaminoglycans.
134. A woman was delivered to a gynecological unit with signs of acute abdomen and suspected extrauterine pregnancy with oviduct rupture. Where will the blood accumulate in this case?

Explanation

In the female lesser pelvis, there are two excavations: the rectouterine pouch and vesicouterine pouch. The rectouterine pouch (pouch of douglas) is an intraperitoneal space between the uterus and the rectum. It is a common place for pelvic fluid or blood from hemorrhage to collect after surgery, or rupture of any etiology.

The vesicouterine pouch is between the urinary bladder anteriorly and the uterus posteriorly. It is a shallower recess (pouch).

135. A patient had a trauma that caused dysfunction of motor centers regulating activity of head muscles. These centers can normally be located in the following area of the cerebral cortex:

Explanation

Primary motor area extends throughout the precentral gyrus and the adjoining lip of central sulcus. Areas 4 and 4S are present here. Muscles of various parts of the body are represented in area 4 in an inverted way from medial to lateral surface. Lower parts of the body are represented in the lateral (upper) surface and upper parts of the body are represented in the lateral (lower) surface. Order of representation from medial (superior, upper) to lateral (inferior, lower) surface: toes, ankle, knee, hip, trunk, shoulder, arm, elbow, wrist, hand, fingers and face. Area 4 is concerned with contraction of discrete muscles.
136. At the post-mortem examination the stomach of a patient with renal failure was found to have a yellow-brown coating on the thickened mucosa. The coating was firmly adhering to its surface and had significant thickness. Microscopy revealed congestion and necrosis of mucosal and submucosal layers, fibrin presence. What is the most likely diagnosis?

Explanation

Fibrinous inflammation is an inflammatory response of mucous surface (oral, respiratory, bowel) to toxins of diphtheria (diphtheric gastritis) or irritant gases. As a result of denudation of epithelium, plasma exudes on the surface where it coagulates and together with necrotized epithelium, forms false membrane that gives this type of inflammation its name. Histologically, fibrin appears as an eosinophilic network of threads or sometimes as an amorphous coagulum.
137. Cystinuria manifests itself in a human as cystine stones in the kidneys (homozygous individuals) or increased cystine content in the urine (heterozygous individuals). Cystinuria is a monogenic disorder. Determine the type of interaction between the genes of cystinuria and normal urine cystine excretion:

Explanation

Semi dominance ( incomplete dominance) is seen when an allele ( a variant) partially dominates the other as seen in the heterozygous case. Complete dominance is seen in the homozygous case. Codominance occurs when the two alleles fully express themselves. Complementarity is seen when two genes contribute in a certain characteristic.
138. A patient with obliterating atherosclerosis has undergone sympathectomy of the femoral artery in the area of femoral triangle. What type of arterial hyperemia was developed in the patient due to this surgery?

Explanation

·        Neuroparalytic arterial hyperemia is caused by damage or blockage of α-adrenoreceptors (sympathetic nervous system). It is characterized by reduction or absence (paralysis) of the sympathetic nervous system effects on the walls of the arteries and arterioles.

·        Neurotonic arterial hyperemia is caused by activation of parasympathetic nervous system; irritation of vascular dilators part of vascular center (CNS) or inhibition of vascular – constrictor part of this center (vasomotor center of CNS); M-cholinoreceptors; H2-histaminereceptors. It is characterized by predominance of the parasympathetic nervous system effects on arterial vascular walls.

139. On bronchoscopy there is a polypoid growth 1,0 cm in diameter with ulcer in its center in the upper lobe of the right lung. Histological investigation revealed a tumor composed of lymphocyte-like cells with hyperchromic nuclei, the cells form layers and bands. What is the most likely tumor type?

Explanation

 

Undifferentiated small cell carcinoma is composed of centrally located small cell lymphocyte-like ( a little larger) with oval hyperchromatic nuclei. In adenocarcinoma, there is a predominance of glandular structure. Squamous cell carcinoma are composed of relatively large cells either keratinizing or non keratinizing.

140. The brain trauma unit received a patient with damaged greater wing of the sphenoid bone. The fracture line crosses the spinous foramen of the sphenoid. What vessel was damaged?

Explanation

The middle meningeal artery and vein, the meningeal branch of the mandibular nerve all pass through the spinous foramen (foramen spinosum) into the cranial cavity.
141. In one of Polessye regions there was an outbreak of helminthiasis manifested by cramps and facial edemas. The developed preventive measures in particular included ban for eating infested pork even after heat processing. What helminthiasis was the case?

Explanation

Trichinosis: causative agent – Trichinella spiralis. Transmission – fecal-oral; undercooked meat (especially pork). Findings: fever, vomiting, nausea, periorbital edema, myalgia. These findings are specific for Trichinosis.
142. A person with vitamin A deficiency develops twilight vision disturbance. Name the cells that fulfill this photoreceptor function:

Explanation

 

The retina contains Rods and Cones cells. The rods are for twilight vision while the cones are for coloured vision.

143. Presented is the biopsy material of an organ consisting of saccule-shaped rounded structures of varying size. Inside these structures there is a gel-like non-cellular substance - colloid; structure walls are composed of one layer of cuboidal cells that lay on the basement membrane. Between the saccules there is connective tissue with vessels. Name this organ:

Explanation

Histologically, the thyroid gland contains follicles, follicular and parafollicular cells. They contain colloidal substance. Thyroxine T4, Triiodothyronine T3 and calcitonin are produced here.
144. A 26-year-old woman with bronchitis has been administered a broad spectrum antibiotic as a causal treatment drug. Specify this drug:

Explanation

The tetracyclines (minocycline, doxycycline) are broad spectrum bacteriostatic antibiotics.

Mechanism of action: entry of these drugs into susceptible organisms is mediated both by passive diffusion and by an energy-dependent transport protein mechanism. The drug binds reversibly to the 30S subunit of the bacterial ribosome, thereby blocking access of the amino acyl-tRNA to the mRNA-ribosome complex at the acceptor site.

Taking these drugs concomitantly with diary foods in the diet decreases absorption due to the formation of non-absorbable chelates of the tetracyclines with calcium ions. Non-absorbable chelates are also formed with other divalent and trivalent cations (e.g. those found in magnesium and aluminum antacids and in Iron preparations).

Adverse effects:

·         Discolouration and hypoplasia of the teeth in growing children

·         Phototoxicity: such as sunburn

·         Dysbacteriosis

·         Gastric discomfort

145. Water affects the mucosa of lower nasal passages resulting in diving reflex. This responce manifests itself as:

Explanation

 

Reflex apnea is an involuntary stop in Breathing as a result of gas or vapor irritation. It is observed in divers as a result of water irritating the nasal mucosa.

Hyperpnea is an increase in depth and rate of breathing and is seen in exercise.

146. Examination of a 56-year-old woman with a history of type 1 diabetes revealed a disorder of protein metabolism that is manifested by aminoacidemia in the laboratory blood test values, and clinically by the delayed wound healing and decreased synthesis of antibodies. Which of the following mechanisms causes the development of aminoacidemia?

Explanation

Type I diabetes: lack of insulin. Normally, insulin stimulates storage of lipids, proteins and glycogen. But in the absence of insulin, glucagon and epinephrine stimulates use fuel reserves through hepatic glycogenolysis, hepatic gluconeogenesis, adipose release of free fatty acids (FFA). ↑in glucagon and epinephrine → ↑protein degradation (muscle) → ↑amino acid → aminoacidemia. Protein degradation = proteolysis.
147. One of the factors that cause obesity is the inhibition of fatty acids oxidation due to:

Explanation

  The major pathway for catabolism of saturated fatty acids is a mitochondrial pathway called β-oxidation. After a long-chain fatty acid (LCFA) enters a cell, it is converted in the cytosol to its Co-A derivative. Because β-oxidation occurs in the mitochondrial matrix, the fatty acid must be transported across inner mitochondrial membrane which is impermeable to Co-A. therefore, a specialized carrier transports the long chain acyl group from the cytosol into the mitochondrial matrix. This carrier is carnitine and this rate-limiting transport process is called the carnitine shuttle. Since carnitine helps the mitochondria utilize energy, it plays a critical role in reducing the occurrence and impact of obesity. In addition to helping the mitochondria burn fat as energy, carnitine is also vital for removing waste products from mitochondria. Obesity and aging contribute to low carnitine levels, which compromises mitochondrial performance and increases insulin resistance, promoting further obesity and carnitine reduction.
148. Streptomycin and other aminoglycosides prevent the joining of formylmethionyl-tRNA by bonding with the 30S ribosomal subunit. This effect leads to disruption of the following process:

Explanation

 

Formyl-methionyl trna is used for the initiation of protein synthesis in bacteria (  Procaryotes) It is not used for protein synthesis in eukaryotes . In gene expression, Translation is a process that occurs in ribosomes present in the endoplasmic reticulum in which proteins are synthesized from RNA molecules.  It occurs after replication and has three phases: initiation, elongation and termination.

Initiation- is the first step in cancer development. Initiators, if not already reactive with DNA, are altered via drug-metabolizing enzymes in the body and are then able to cause changes in DNA (mutations).

Promotion- Once a cell has been mutated by an initiator, it is susceptible to the effects of promoters. These compounds promote the proliferation of the cell, giving rise to a large number of daughter cells containing the mutation created by the initiator.

Progression- refers to the stepwise transformation of a benign tumor to a neoplasm and to malignancy.  An increased growth rate, invasiveness, metastasis and an alteration in biochemistry and morphology are present in this phase.

149. In hot weather ventilators are often used to normalize the microclimate in the heated rooms. It leads to intensified heat transfer from the human body by means of:

Explanation

Convection is a way the body eliminates heat by means of transferring heat via moving particles of air or water. To dissipate heat by means of convection, body surface shall be airflowed at a temperature that is lower than the temperature of the skin. At that, air layer contacting with the skin warms up, decreases its density, rises and is replaced by cooler, denser air. By increasing the speed of the air flow (wind, ventilation) heat emission increases significantly as well (forced convection). Convection requires convection current; current of gases or liquids (Key words: air over exposed area of skin).

The areas of the skin covered with clothes has a higher temperature because the convection current does not have direct access to the skin in that area as compared to the exposed area of the skin.

Heat Radiation is a way the surface of the human body emits heat to the environment in the form of infrared rays. The amount of heat the body radiates to the environment is proportional to the surface of radiation area and to the difference between the mean values of skin and environment temperature. The surface radiation area is the total surface area of body parts that contact the air. Elimination of heat by radiation increases with a decrease in ambient temperature and decreases with its increase. It is possible to reduce elimination of heat by radiation via reduction of the surface of radiation area (“winding oneself into a ball”). Heat radiation does not require a medium for transfer of heat. (Key words: naked or lightly clothed).

Evaporation is a way the body dissipates heat to the environment by its evaporation via sweat or evaporation of moisture from the skin and respiratory tract mucous membranes of (“wet” heat loss). Evaporation closely related to relative humidity.

Conduction is a way the body eliminates heat by means of direct contact with another object. Heat is transferred down the temperature gradient (i.e. from the object of higher temperature to the object of lower temperature). Conduction requires contact with another object (Key words:  in water).

150. During gastric resection the patient received mixed anesthesia with tubocurarin chloride muscle relaxant; to restore spontaneous respiration the patient received proserin. What pharmacological group does this drug belong to?

Explanation

 

Proserin is an anticholinesterase drug. It decreases the breakdown of acetylcholine

Angiotensin converting enzyme inhibitors - Captopril, enalapril.

Calcium blockers- verapamil.

Muscarinic antagonist- atropine.

151. A 67-year-old man consumes eggs, pork fat, butter, milk and meat. Blood test results: cholesterol - 12,3 mmol/l, total lipids - 8,2 g/l, increased low-density lipoprotein fraction (LDL). What type of hyperlipoproteinemia is observed in the patient?

Explanation

Type IIa (familial hyperlipoproteinemia: ↑LDL and cholesterol. Autosomal dominant; due to absent or defective LDL receptors.

Type I:  ↑chylomicrons, triacylglycerol (TAG), cholesterol. Autosomal recessive; due to lipoprotein lipase deficiency or altered apolipoprotein C-II.

Type IV: ↑very low density lipoprotein (VLDL) and TAG. Autosomal dominant; due to hepatic overproduction of VLDL.

Type IIb: similar to Type IIa, except that VLDL is also increased and VLDL is normal for IIa.

152. To treat bronchitis the patient was prescribed a beta-lactam antibiotic. Its mechanism of action is based on inhibition of murein production, which results in death of the causative agent. Name this drug:

Explanation

Beta lactam is a group of antibiotics that destroy bacterial cell wall. They do this by inhibiting Transpeptidase enzyme ( responsible for cross linking of fibres in bacterial cell wall). To this group belong Penicillins, Cephalosporins, Monobactam and carbapenems.

 
153. Cell membrane rest potential changed from -85 to -90 mV. It can be caused by activation of the following cell membrane channels:

Explanation

 Depolarization- sodium influx to the cell. Repolarization- Potassium efflux. A change in the resting membrane potential from -85mv to -90mv indicates repolarization.

 
154. Ionizing radiation or vitamin E deficiency affect the cell by increasing lysosome membrane permeability.What are the possible consequences of this pathology?

Explanation

Vitamin E is an antioxidant responsible for killing Reactive oxygen species ( bad oxygen). These ROS destroy membrane of cells leading partial or complete cell destruction.
155. A 22-year-old woman ate some seafood. 5 hours later her torso and distal parts of her limbs developed small itchy papules which were partially fused together. One day later the rash disappeared spontaneously. Specify the hypersensitivity mechanism underlying these changes:

Explanation

It is anaphylactic because of the relatively short time it took to manifest the symptoms. It is local because, it affects just the trunk and distal part of limb. Systemic will have systemic effects like anxiety, loss of consciousness, shock, difficulty in breathing.

Type I Hypersensitivity reaction (HSR); anaphylactic and atopic: free antigen cross-links IgE on presensitized (i.e. exposed to the antigen before) mast cells and basophils, triggering immediate release of vasoactive amines that act at postcapillary venules (i.e. histamine). Reaction develops rapidly after antigen exposure because of preformed antibody from first exposure. IgE is the main immunoglobulin involved in type I HSR. Type I: uses IgE and IgG4

156. During acute hemorrhage the body loses not only fluid but also electrolytes. What substance solution can be used as a simple blood substitute?

Explanation

A blood substitute is a substance used in place of blood to carry out the same function. Example- Polyethylene glycol. Sodium Chloride is used for replacement of electrolytes.
157. A patient has been admitted to the contagious isolation ward with signs of jaundice caused by hepatitis virus. Which of the symptoms given below is strictly specific for hepatocellular jaundice?

Explanation

In viral hepatitis, there is generalized liver dysfunction involving uptake and conjugation of unconjugated bilirubin, secretion of conjugated bilirubin into bile ducts, and recycling of urobilinogen. Alanine transaminase (ALT) and Aspartate transaminase (AST) are increased (↑), but ALT is higher than AST and there is a slight ↑ in alkaline phosphatase (ALP) and ɣ-glutamyltransferase (GGT).

ALT is a specific enzyme for liver necrosis; present in the cytosol. ALT>AST: viral hepatitis.

AST is present in the mitochondria. Alcohol damages mitochondria AST>ALT indicates alcoholic hepatitis

158. In a body of a 37-year-old woman, who died with signs of pulmonary edema, there was detected acute deformation of the aortic valve: it is shortened, thickened, ulcerated, has areas of stone-like density. On its external surface there are large, up to 2 cm in diameter, thrombotic plaques. Left ventricle wall is 2,2 cm thick. Cardiac muscle is dull, matt, and flaccid. What type of endocarditis corresponds with described alterations of the aortic valve?

Explanation

Polypous ulcerative endocarditis is a morphological form of septic (bacterial) endocarditis. It develops on both sclerotic and intact valves. Large thromboembolic polyp-shaped plaques appear on sclerotic valves. The plaques are easily crumbled and are saturated with calcium which is characteristic for this pathology. After removal of the plaques, ulcerative defects are seen on the sclerotic and deformed cusps of the valves.
159. Students study the stages of gametogenesis. They analyze a cell with haploid number of chromosomes, whith each chromosome consisting of two chromatids. The chromosomes are located in the equatorial plane of the cell. Such situation is typical of the following stage of meiosis:

Explanation

Metaphase: chromosomes are located on equatorial plane; fully condensed chromosomes; in this phase we study the cell’s karyotype. Metaphase of second division: haploid number of chromosomes
160. A 38-year-old woman developed a bronchial asthma attack. Which of the listed bronchial spasmolytics is effective for emergency aid and belongs to beta-2- adrenergic agonists?

Explanation

 

Salbutamol belongs to the group of Beta 2 agonists. It causes Bronchodilation. Adrenaline acts on alpha receptors. Atropine is a muscarinic blocker.

161. After emotional upset a woman has been suffering from disturbed sleep for several days. What soporific drug would be preferable for this type of insomnia?

Explanation

All of the benzodiazepines used to treat anxiety (tranquilizers) have some sedative properties and some can produce hypnosis (artificially produced sleep) at higher doses. Their effects have been shown to be mediated by the α1-GABAA receptors. Benzodiazepines are the most widely used anxiolytic drugs e.g. Diazepam, lorazepam, nitrazepam, clonazepam etc. They are also used to treat neurosis.
162. During training session in the laboratory the students were performing spirography on themselves. What indicator CANNOT be measured with this method?

Explanation

 spirometry is a test for pulmonary function. It measures;

  1. Vital Capacity: The volume of air breathed out after maximum inhalation.

  2. Forced vital capacity And forced Expiratory volume etc

 

N/B Functional residual capacity- amount of air in the lungs after respiration cannot be measured using a spirometer.

163. In the South and Central America there can be found a species of trypanosomes that is the causative agent of Chagas disease. What animal is the infection carrier specific to this disease?

Explanation

 Chagas disease ( american trypanosomiasis) is caused by Trypanosoma cruzi which is transmitted by Triatomine bug. Mosquito transmits malaria. Tsetse flies transmit african trypanosomiasis. 

 
164. A 54-year-old woman was brought to the emergency department after a car accident. A traumatologist diagnosed her with multiple fractures of the lower extremities. What kind of embolism is the most likely to develop in this case?

Explanation

Embolism is occlusion of a vessel by material travelling in the circulation.

* Fat/Adipose embolism: obstruction of arterioles and capillaries by fat globules constitutes fat embolism. It may occur following severe fracture trauma to bones, inflammation of bones and soft tissues, fatty liver

* Thromboembolism: a detached thrombus or part of thrombus which may arise in the arterial or venous circulation.

* Gas embolism: two main forms of gas embolism are air embolism and decompression sickness. Air embolism is usually due to accidental pumping of air into the venous circulation during intravenous (IV) injection or transfusion ( bubble – air escaped).

Tissue embolism: fragments of tissue.

165. A 30-year-old man with diabetes mellitus type I was hospitalized. The patient is comatose. Laboratory tests revealed hyperglycemia and ketonemia. What metabolic disorder can be detected in this patient?

Explanation

During insulin deficiency, glucose cannot be utilized by the peripheral tissues for energy. So, a large amount of fat is broken down to release energy. It causes the formation of excess ketone bodies leading to acidosis. One more reason for acidosis is that the ketone bodies are excreted in combination with sodium ions through urine (ketonuria). Sodium exchanged H+, which diffuse from the renal tubules into ECF adding to acidosis.  In diabetic patients, the body produces too much acid & the kidneys are not removing them fast enough as they are produced which results in metabolic acidosis.
166. Typical manifestations of food poisoning caused by C. botulinum are double vision, abnormal functioning of the swallowing and breathing. These symptoms develop as the result of:

Explanation

Clostridium botulinum is a gram positive rod (bacillus). They secrete botulinum toxin (an exotoxin). Findings: flaccid paralysis, floppy baby; toxin prevents release of stimulatory (acetylcholine) signals at neuromuscular junctions → flaccid paralysis. Exotoxin is secreted from certain species of gram positive and gram negative bacteria. Its toxoids are used as vaccines. Inhibition of the release of stimulatory neurotransmitters can lead to the symptoms listed in the question.
167. An infant, who was on synthetic formula feeding, developed signs of vitamin B1 deficiency. What reactions does this vitamin take part in?

Explanation

Thiamine pyrophosphate (TPP, or thiamine diphosphate, TDP) is the active form of the vitamin thiamine. TPP is an important cofactor that acts catalytically in the decarboxylation of α-keto acids and the transketolase reaction.

 
168. A 30-year-old woman developed the signs of virilism (body hair growth, balding temples, disturbed menstrual cycle). What hormone can cause this condition when hyperproduced?

Explanation

These symptoms indicates an increase in male sex hormones. This can probably be a pathology of the adrenals or ovaries in a female, which leads to increase production of testosterone (the only male sex hormone in the options)
169. A histological specimen shows signifi- cant amount of mucous connective tissue (Wharton’s jelly), vessels, as well as residual yolk and allantois. Name this organ:

Explanation

 

Histologically, the umbilical cord contains two arteries and one vein. It contains wharton’s jelly which is a gelatinous substance found within.

170. On examination of a 6-year-old child the doctor noticed grayish film on the child’s tonsils. Microscopy of the smears stained by Neisser method detected there Corynebacterium diphtheriae. What morphologic feature was the most indicative for determining the type of the agent?

Explanation

 

Volutin granules are storage granules found inside the cytoplasm, they are found in Corynebacterium diphtheriae when stained with Neisser/Leffler’s method and are key points in diagnosis.

171. During the sports competition a boxer received a strong blow to the abdomen, which caused a knockout due to a brief drop in blood pressure. What physiological mechanisms are the cause of this condition?

Explanation

Parasympathetic nervous system when stimulated leads to: a decrease in blood pressure, heart rate and breathing rate, mydriasis, increase in GIT functions etc

 

Sympathetic activation : an increase in heart rate, blood pressure and breathing rate, miosis and decrease in GIT activities.

172. After a severe stress the patient presents with eosinopenia in the blood test. In this case the decreased number of eosinophils can explain changes in the level of the following hormones:

Explanation

Glucocorticoids have anti inflammatory, antipyretic and immunosuppressive functions. A decrease in eosinophils indicates an immunosuppression.
173. A 30-year-old patient’s blood test revealed the following: erythrocyte count is 6 · 1012/l, hemoglobin is 10,55 mmol/l. Vaquez’s disease was diagnosed. Name the leading part of pathogenesis in this case:

Explanation

Hemoblastoses refer to neoplastic clonal diseases, arising from hemopoietic organs. Erythrocytes which originate from the hemopoietic organ can be greatly increased (erythrocytosis) in myeloproliferative disorders. Myeloproliferative disorders includes chronic myeloleukemia, polycythemia vera (erythemia, Vaquez disease, Osler’s disease, osler-vaquez disease) etc. Vaquez disease is a primary form of erythrocytosis.
174. Deaf parents with genotypes DDee and ddEE gave birth to a child with normal hearing. Specify the interaction of D and E genes:

Explanation

 Semi dominance ( incomplete dominance) is seen when an allele ( a variant) partially dominates the other as seen in the heterozygous case. Complete dominance is seen in the homozygous case. Complementarity is seen when two genes contribute in a certain characteristic. Polymery is when two or more genes affect one phenotype.

 
175. Corticosteroid hormones regulate the adaptation processes of the body as a whole to environmental changes and ensure the maintenance of internal homeostasis. What hormone activates the hypothalamopituitary-adrenal axis?

Explanation

Liberins and statins are groups of hormones released from the hypothalamus that control the release of stimulating hormone from the pituitary gland. Eg, Corticoliberin stimulates the release of Adrenocorticotropic hormone (ACTH) from the anterior pituitary gland (adenohypophysis). ACTH then controls the release of adrenal cortex hormones.

 
176. A patient with signs of emotional lability that result in troubled sleep has been prescribed nitrazepam. Specify the sleepinducing mechanism of this drug:

Explanation

Nitrazepam is a hypnotic (tranquilizer) drug from the benzodiazepine family. Other drugs in this family include diazepam, lorazepam, triazolam, oxazepam etc. they facilitate GABAA action by increasing frequency of Cl- channel opening. They decrease rapid eye movement (REM) sleep.
177. A 50-year-old inpatient during examination presents with glucosuria and blood glucose of 3,0 mmol/l, which are the most likely to be caused by:

Explanation

This patient presents with glycosuria and hypoglycemia which indicates that glucose is being filtered out. Pathology arises from the renal corpuscle. Eg, Glomerulonephritis.

 
178. A man is suffering from diarrhea. In summer he spent his vacation in the south at the sea coast. Bacteria with the following properties were detected in his feces: gram negative curved mobile monotrichous bacilli that do not produce spores or capsules. They are undemanding to nutrient medium but require alkaline reaction (рН - 8,5-9,5). Described are the agents of the following enteric infection:

Explanation

Vibrio cholera is the cause of cholera. It is curved, comma-shaped, gram negative rods. It is transmitted by fecal contamination of water and food, primarily from human sources. Clinically, watery diarrhea in large volumes is the hallmark of cholera. There are no red blood cells or white blood cells in the stool. Rice-water stool is the term often applied to the non-bloody effluent. Grows in an alkaline media. Dark-field and phase contrast microscopy have been used for screening fecal specimens for the presence of V. cholera. With these techniques, liquid stools are microscopically examined for the presence of organisms with typical darting (“shooting star”) mobility [analogous to ‘hanging drop’].
179. A pregnant woman was detected to have IgM to rubella virus. An obstetrician-gynecologist recommended therapeutic abortion due to the high risk of teratogenic affection of the fetus. Detection of IgM was of great importance as it is these specific immunoglobulins that:

Explanation

High IgM level usually indicate an acute or primary infection because, it is the first antibody to appear in response to initial exposure to an antigen. Secondary or reinfection show an increase in IgG. IgM antibodies appear early in the course of an infection, this makes it useful in the diagnosis of infectious diseases. Demonstrating IgM antibodies in a patient’s serum indicates recent infection or in a neonate’s serum, it indicates intrauterine infection.
180. During examination of a teenager with xanthomatosis the family history of hypercholesterolemia is revealed. What transportable lipids are increased in concentration in case of such a disease?

Explanation

 

xanthomatosis is characterised by the deposition of cholesterol rich contents in various parts of the body eg. skin. It is related to familial hypercholesterolemia in which there is increased LDL levels.

181. On examination the patient is found to have low production of adrenocorticotropic hormone. How would this affect production of the other hormones?

Explanation

Adrenocorticotropic hormone is a hormone produced by the anterior pituitary gland ( adenohypophysis). It stimulates the production of hormones from the adrenal cortex.

 
182. Parkinson’s disease is caused by disturbance of dopamine synthesis. What brain structure synthesizes this neurotransmitter?

Explanation

Parkinson disease is associated with a loss of dopaminergic neurons in the substantia nigra. Parkinsonism is a progressive neurological disorder of muscle movement, characterized by tremors, muscular rigidity, bradykinesia (slowness in initiating and carrying out voluntary movements) and postural and gait abnormalities. Most cases involve people over the age of 65.

The disease is correlated with destruction of dopaminergic neurons in the substantial nigra with a consequent reduction of dopamine actions in the corpus striatum, parts of the brain’s basal ganglia system that are involved in motor control.

The substantial nigra, part of the extrapyramidal system is the source of dopaminergic neurons that terminate in the neostriatum.

Treatment: drugs – levodopa, carbidopa, selegiline, rasagiline

So far, levodopa has been the only drug tested on Parkinson.

183. Determining a patient’s blood group with monoclonal test-reagents revealed positive agglutination reaction to anti-A and anti-B reagents, and negative reaction to anti-D. What blood group does this patient have?

Explanation

Blood group O(I): no antigens, therefore no agglutination.

Blood group A(II): A antigen, agglutinate with blood group B(III) and O(I).

Blood group B(III): B antigen, agglutinate with blood group A(II) and O(I).

Blood group AB(IV): A and B antigen, agglutinate with all blood groups. No antibody.

Blood group name is determined by the antigen present on RBC, but the patient has an opposite antibody. So whenever, the antibody corresponds to the antigen, there is agglutination. Since O does not have any antigen, no agglutination can occur.

184. A patient visited a dentist to extract a tooth. After the tooth had been extracted, bleeding from the tooth socket continued for 15 minutes. Anamnesis states that the patient suffers from active chronic hepatitis. What phenomenon can extend the time of hemorrhage?

Explanation

Substances necessary for coagulation or clotting of blood are called clotting factors. Fibrinogen is the factor I of the thirteen clotting factors. Most of the clotting factors are proteins in the form of enzymes synthesized in the liver.

In chronic hepatitis (i.e. relatively prolonged course of inflammation of the liver), the protein-synthetic function of the liver is impaired. Meaning, there is a marked decrease of clotting factors which results in uncontrolled and protracted hemorrhage.

185. During ascent into mountains a person develops increased respiration rate and rapid heart rate. What is the cause of these changes?

Explanation

 

In high grounds ( mountains), there is a decrease in partial pressure of O2, this activates chemoreceptors found on the aortic arch and carotid sinus which stimulates an increase in depth and rate of breathing as well as heart rate.

186. To stop the bleeding the patient was prescribed a direct coagulant. During introduction of the solution the patient was complaining of pain along the vein, hot sensation, and palpitations. Name the drug that causes such symptoms:

Explanation

Calcium chloride is a drug indicated in hypocalcemic tetany, overdose of magnesium sulfate etc. It increases heart contractility. Streptokinase is a thrombolytic. Pentoxyl reduces muscle pain and improves blood flow rate.
187. A student, whose educational achievements throughout the semester were poor, feels emotionally tense during the final test. What is the primary cause that induced leading mechanism of emotional tension in this case?

Explanation

Due to his poor studies, a lack of information in the exams activated his sympathetic nervous system leading to his tensed state.
188. A young family came for a genetic counseling to identify the father of their child. The husband insists that the child does not resemble him at all and cannot possibly be his. Polymerase chain reaction method for person identification is based on the following:

Explanation

Polymerase chain reaction (PCR) is a molecular biology laboratory procedure used t amplify a desired fragment of DNA. Useful as a diagnostic tool (e.g. HIV, herpes encephalitis).

Nucleic acid-based (DNA or RNA based) tests are highly specific, quite sensitive (especially the amplification tests) and mush faster than culturing the organism. Nucleic acid amplification tests utilize the PCR or other amplifying processes to increase the number of specific DNA or RNA molecules so the sensitivity of the test is significantly higher than that of unamplified tests. Steps in PCR includes: denaturation; annealing and elongation. These steps are repeated multiple times for DNA sequence amplification.

189. A 52-year-old man presents with fever and pain in the joints. Both of his first metatarsophalangeal articulations are deformed, swollen, and reddened. Blood urea is high. The patient is diagnosed with gout. What is the main developmental factor in the pathogenesis of this disease?

Explanation

  This is a classic description of gout, which is characterized by hyperuricemia. The end product of the purine nucleotides catabolism in humans and other primates is uric acid (urate) which is excreted in urine. Allopurinol and febuxostat inhibits Xanthine oxidase (XO). Hypoxanthine and Xanthine which is more soluble is excreted in urine. Purine nucleotides (adenine and guanine).  AMP – Adenosine monophosphate; GMP – Guanosine monophosphate
190. A 67-year-old man was delivered to the cardiology unit with complaints of periodical pain in the heart, dyspnea after even insignificant physical exertion, cyanosis, and edemas. ECG revealed additional contractions of the heart ventricles. Name this type of rhythm disturbance:

Explanation

Extrasystole is the premature contraction (an extra contraction) of the heart before its normal contraction. In other words, it is an extra contraction of the heart before its normal contraction. It is caused by an ectopic focus (discharge of an impulse from any part of the heart other than the Sinoatrial node).

Bradycardia: ↓heart rate; Tachycardia: ↑heart rate; Flutter: rapid heart contractions; Fibrillation: very rapid heart contractions. (In all this, there is no extra heart contraction, we just have faster contractions).

191. During narcosis the patient developed a risk of cerebral edema. What drug should be administered in this case?

Explanation

Bumetanide, furosemide, torsemide, ethacrynic acid are diuretics that have their major action on the ascending limb of the loop of henle (loop diuretics). These drugs are useful in emergency situation which calls for a rapid, intense dieresis (forced dieresis). Loop or high-ceiling diuretics inhibit the cotransport of Na+/K+/2Cl- on the luminal membrane in the ascending limb of the loop of henle.  

Phenazepam is a tranquilizer while dopamine is a neurotransmitter produced in the substantia nigra in the brain.

192. A patient with pulmonary tuberculosis is prescribed the most effective antituberculous antibiotic. Name this drug:

Explanation

 Drugs used in the treatment of tuberculosis and their side effects include:

  1. Rifampicin- red urine and body fluids

  2. Isoniazid- neuritis ie, numbness of feet etc

  3. Pyrazinamide- gout, liver toxicity

  4. Ethambutol- optic neuritis and kidney related issues

  5. Streptomycin-  ototoxicity

Isoniazid and rifampicin are the most active anti-tuberculosis drugs.

 
193. During surgery performed in the abdominal cavity a surgeon located ligament of liver stretching from anterior abdominal wall (navel) to inferior surface of liver. What ligament is it?

Explanation

Round ligament of liver (ligamentum teres hepatis): it runs from the visceral surface to the navel; it is an obliterated peritoneum–enfolded umbilical vein. The orifice of the vein may remain partially patent (open) and thus the vein can be used for infusion of drugs and radiopaque agents. The umbilical vessels closure occurs during the first week of life; the umbilical vein transforms into peritoneum–enfolded round ligament of liver. The umbilical arteries also close and transform into the medial umbilical ligaments. The single umbilical vein is responsible for carrying oxygenated blood from placenta to fetus in fetal circulation.
194. On examination of a patient with disease onset 5 days ago the doctor suspected tularemia and prescribed the patient tularin intracutaneously. What is the purpose of this drug administration in the patient?

Explanation

 

Drugs are firstly given intracutaneously and observed for few minutes to see if there be any reaction ( formation of a bleb). This is to confirm that the patient isn't allergic to the drug to be administered.

195. A patient developed increased blood content of HCO− 3 against the background of repeated and uncontrollable vomiting. What will be the leading mechanism in compensation of developed acid-base imbalance?

Explanation

Repeated vomiting led to the accumulation of bicarbonates which causes alkalosis. To correct this, the body will have to decrease pulmonary ventilation ie, reduce O2 amount and increase CO2 amount in circulation leading to the accumulation of carbonic acid (acidosis), compensating the earlier state.

 
196. A 13-year-old boy presents with eczematous rashes on his shins and torso. Anamnesis states cases of otitis, pneumonia, and furuncles in the patient. Blood test: platelets - 70 · 109/l, low activity of T helper and T suppressor cells, low IgM, with normal IgA and IgG. What immunodeficient disease does this boy have?

Explanation

Wiskott aldrich syndrome is an X-linked recessive disorder characterised by eczema, thrombocytopenia and low level of IgM immunoglobulin.

Chediak Higashi disease is a disorder in phagocytosis leading to recurrent pyogenic infections, albinism and peripheral neuropathy.

In severe combined immunodeficiency, both B and T cells are deficient.

197. During the exam a student was unable to correctly answer all the questions in his question card, which was accompanied by the reddening of his face and hot sensation. What type of arterial hyperemia did the student develop in this case?

Explanation

 

Neuroparalytic form is seen in Sympathectomy (cutting) of the α adrenergic fibres and nerves. Neurotonic is seen in stimulation ( increased tone) of these fibres.

198. An injured person with wound of the anterior cervical region presents with hemorrhage. The outflowing blood is dark. What vessel is damaged?

Explanation

   

Dark blood indicates Venous blood, bright red- arterial. The anterior cervical region is the Area of the neck bounded by the mandible, anterior border of the sternocleidomastoid muscle, and the anterior midline of the neck. The anterior jugular vein lies here.

199. Due to trauma the patient presents with disturbed function of the parotid gland. What nerve ensures its secretion function?

Explanation

 

The petrous minor nerve supplies the parotid gland, the petrous major nerve supplies the submandibular and sublingual glands.

200. A patient developed pyoinflammatory process of periodontal tissues caused by activation of the microorganisms inherent in the body, which are a part of oral mucosal microflora. What type of infection is it?

Explanation

 An autoinfection is one that is caused by the normal bacteria flora present in our body. An Exogenous infection is caused  by microorganisms from our environment. In co-infection, the individual has two seperate infections eg TB and typhoid. A mixed infection is one which is caused by many microorganisms. A superinfection is an infection imposed while another infection is in a period of treatment